The most reliable index to determine the respiratory status of a client is to

– Questions and choices are randomly arranged, the answer is revealed instantly after each question, and there is no time limit for the exam.

MSN Exam for Asthma and COPD (PM)*

Please wait while the activity loads.
If this activity does not load, try refreshing your browser. Also, this page requires javascript. Please visit using a browser with javascript enabled.

If loading fails, click here to try again

Choose the letter of the correct answer. Good luck!

Start

Congratulations - you have completed MSN Exam for Asthma and COPD (PM)*. You scored %%SCORE%% out of %%TOTAL%%. Your performance has been rated as %%RATING%%

Your answers are highlighted below.

Question 1

Immediately following a thoeacentesis, which clinical manifestations indicate that a complication has occurred and the physician should be notified?

A

Serosanguineous drainage from the puncture site

B

Increased temperature and blood pressure

C

Increased pulse and pallor

D

Hypotension and hypothermia

Question 1 Explanation: 

Increased pulse and pallor are symptoms associated with shock. A compromised venous return may occur if there is a mediastinal shift as a result of excessive fluid removal. Usually no more than 1 L of fluid is removed at one time to prevent this from occurring.

Question 2

Auscultation of a client’s lungs reveals crackles in the left posterior base. The nursing intervention is to:

A

Repeat auscultation after asking the client to deep breathe and cough.

B

Instruct the client to limit fluid intake to less than 2000 ml/day.

C

Inspect the client’s ankles and sacrum for the presence of edema

D

Place the client on bedrest in a semi-Fowlers position.

Question 2 Explanation: 

Although crackles often indicate fluid in the alveoli, they may also be related to hypoventilation and will clear after a deep breath or a cough. It is, therefore, premature to impose fluid (B) or activity (D) restrictions . Inspection for edema (C) would be appropriate after reauscultation.

Question 3

The nurse assesses the respiratory status of a client who is experiencing an exacerbation of COPD secondary to an upper respiratory tract infection. Which of the following findings would be expected?

A

Normal breath sounds

B

Prolonged inspiration

C

Normal chest movement

D

Coarse crackles and rhonchi

Question 3 Explanation: 

Exacerbations of COPD are frequently caused by respiratory infections. Coarse crackles and rhonchi would be auscultated as air moves through airways obstructed with secretions. In COPD, breath sounds are diminished because of an enlarged anteroposterior diameter of the chest. Expiration, not inspiration, becomes prolonged. Chest movement is decreased as lungs become overdistended.

Question 4

A nurse is caring for a client hospitalized with acute exacerbation of COPD. Which of the following would the nurse expect to note on assessment of this client?

A

Increased oxygen saturation with exercise

B

Hypocapnia

C

A hyperinflated chest on x-ray film

D

A widened diaphragm noted on chest x-ray film

Question 4 Explanation: 

Clinical manifestations of COPD include hypoxemia, hypercapnia, dyspnea on exertion and at rest, oxygen desaturation with exercise, and the use of accessory muscles of respiration. Chest x-ray films reveal a hyperinflated chest and a flattened diaphragm is the disease is advanced.

Question 5

A client has been taking benzonatate (Tessalin Perles) as prescribed. A nurse concludes that the medication is having the intended effect if the client experiences:

A

Decreased anxiety level

B

Increased comfort level

C

Reduction of N/V

D

Decreased frequency and intensity of cough

Question 5 Explanation: 

Benzonatate is a locally acting antitussive the effectiveness of which is measured by the degree to which it decreases the intensity and frequency of cough without eliminating the cough reflex.

Question 6

The nurse reviews the ABG values of a client. The results indicate respiratory acidosis. Which of the following values would indicate that this acid-base imbalance exists?

A

pH of 7.48

B

PCO2 of 32 mm Hg

C

pH of 7.30

D

HCO3– of 20 mEq/L

Question 7

A nurse is suctioning fluids from a client through an endotracheal tube. During the suctioning procedure, the nurse notes on the monitor that the heart rate decreases. Which of the following is the most appropriate nursing intervention?

A

Continue to suction

B

Ensure that the suction is limited to 15 seconds

C

Stop the procedure and reoxyenate the client

D

Notify the physician immediately.

Question 7 Explanation: 

During suctioning, the nurse should monitor the client closely for side effects, including hypoxemia, cardiac irregularities such as a decrease in HR resulting from vagal stimulation, mucousal trauma, hypotension, and paroxysmal coughing. If side effects develop, especially cardiac irregularities, this procedure is stopped and the client is reoxygenated.

Question 8

When teaching a client with COPD to conserve energy, the nurse should teach the client to lift objects:

A

While inhaling through an open mouth.

B

While exhaling through pursed lips

C

After exhaling but before inhaling.

D

While taking a deep breath and holding it.

Question 8 Explanation: 

Exhaling requires less energy than inhaling. Therefore, lifting while exhaling saves energy and reduced perceived dyspnea. Pursing the lips prolongs exhalation and provides the client with more control over breathing. Lifting after exhalation but before inhaling is similar to lifting with the breath held. This should not be recommended because it is similar to the Valsalva maneuver, which can stimulate cardiac dysrhythmias.

Question 9

Which of the following types of asthma involves an acute asthma attack brought on by an upper respiratory infection?

A

Emotional

B

Extrinsic

C

Intrinsic

D

Mediated

Question 9 Explanation: 

Intrinsic asthma doesn’t have an easily identifiable allergen and can be triggered by the common cold. Asthma caused be emotional reasons is considered to be in the extrinsic category. Extrinsic asthma is caused by dust, molds, and pets; easily identifiable allergens. Mediated asthma doesn’t exist.

Question 10

A client has an order to have radial ABG drawn. Before drawing the sample, a nurse occludes the:

A

Brachial and radial arteries, and then releases them and observes the circulation of the hand.

B

Radial and ulnar arteries, releases one, evaluates the color of the hand, and repeats the process with the other artery.

C

Radial artery and observes for color changes in the affected hand.

D

Ulnar artery and observes for color changes in the affected hand.

Question 10 Explanation: 

Before drawing an ABG, the nurse assesses the collateral circulation to the hand with Allen’s test. This involves compressing the radial and ulnar arteries and asking the client to close and open the fist. This should cause the hand to become pale. The nurse then releases pressure on one artery and observes whether circulation is restored quickly. The nurse repeats the process, releasing the other artery. The blood sample may be taken safely if collateral circulation is adequate.

Question 11

Which of the following diets would be most appropriate for a client with COPD?

A

Low fat, low cholesterol

B

Bland, soft diet

C

Low-Sodium diet

D

High calorie, high-protein diet

Question 11 Explanation: 

The client should eat high-calorie, high-protein meals to maintain nutritional status and prevent weight loss that results from the increased work of breathing. The client should be encouraged to eat small, frequent meals. A low-fat, low-cholesterol diet is indicated for clients with coronary artery disease. The client with COPD does not necessarily need to follow a sodium-restricted diet, unless otherwise medically indicated.

Question 12

A nurse is caring for a client with emphysema. The client is receiving oxygen. The nurse assesses the oxygen flow rate to ensure that it does not exceed

A

1 L/min

B

2 L/min

C

6 L/min

D

10 L/min

Question 12 Explanation: 

One to 3 L/min of oxygen by nasal cannula may be required to raise to PaO2 to 60 to 80 mm Hg. However, oxygen is used cautiously and should not exceed 2 L/min. Because of the long-standing hypercapnia, the respiratory drive is triggered by low oxygen levels rather than increased carbon dioxide levels, as is the case in normal respiratory system.

Question 13

It’s highly recommended that clients with asthma, chronic bronchitis, and emphysema have Pneumovax and flu vaccinations for which of the following reasons?

A

All clients are recommended to have these vaccines

B

These vaccines produce bronchodilation and improve oxygenation.

C

These vaccines help reduce the tachypnea these clients experience.

D

Respiratory infections can cause severe hypoxia and possibly death in these clients.

Question 13 Explanation: 

It’s highly recommended that clients with respiratory disorders be given vaccines to protect against respiratory infection. Infections can cause these clients to need intubation and mechanical ventilation, and it may be difficult to wean these clients from the ventilator. The vaccines have no effect on bronchodilation or respiratory care.

Question 14

The term “blue bloater” refers to which of the following conditions?

A

Adult respiratory distress syndrome (ARDS)

B

Asthma

C

Chronic obstructive bronchitis

D

Emphysema

Question 14 Explanation: 

Clients with chronic obstructive bronchitis appear bloated; they have large barrel chests and peripheral edema, cyanotic nail beds and, at times, circumoral cyanosis. Clients with ARDS are acutely short of breath and frequently need intubation for mechanical ventilation and large amounts of oxygen. Clients with asthma don’t exhibit characteristics of chronic disease, and clients with emphysema appear pink and cachectic (a state of ill health, malnutrition, and wasting).

Question 15

A nurse teaches a client about the use of a respiratory inhaler. Which action by the client indicated a need for further teaching?

A

Removes the cap and shakes the inhaler well before use.

B

Presses the canister down with finger as he breathes in.

C

Inhales the mist and quickly exhales.

D

Waits 1 to 2 minutes between puffs if more than one puff has been prescribed.

Question 15 Explanation: 

The client should be instructed to hold his or her breath at least 10 to 15 seconds before exhaling the mist.

Question 16

The nurse is teaching the client how to use a metered dose inhaler (MDI) to administer a Corticosteroid drug. Which of the following client actions indicates that he us using the MDI correctly? Select all that apply.

A

The inhaler is held upright.

B

Head is tilted down while inhaling the medication

C

Client waits 5 minutes between puffs.

D

Mouth is rinsed with water following administration

E

Client lies supine for 15 minutes following administration.

Question 17

A nurse plans care for a client with chronic obstructive pulmonary disease, knowing that the client is most likely to experience what type of acid-base imbalance?

A

Respiratory acidosis

B

Respiratory alkalosis

C

Metabolic acidosis

D

Metabolic alkalosis

Question 17 Explanation: 

Respiratory acidosis is most often due to hypoventilation. Chronic respiratory acidosis is most commonly caused by COPD. In end-stage disease, pathological changes lead to airway collapse, air trapping, and disturbance of ventilation-perfusion relationships.

Question 18

Which of the following respiratory disorders is most common in the first 24 to 48 hours after surgery?

A

Atelectasis

B

Bronchitis

C

Pneumonia

D

Pneumothorax

Question 18 Explanation: 

Atelectasis develops when there’s interference with the normal negative pressure that promotes lung expansion. Clients in the postoperative phase often splint their breathing because of pain and positioning, which causes hypoxia. It’s uncommon for any of the other respiratory disorders to develop.

Question 19

A client with allergic rhinitis asks the nurse what he should do to decrease his symptoms. Which of the following instructions would be appropriate for the nurse to give the client?

A

“Use your nasal decongestant spray regularly to help clear your nasal passages.”

B

“Ask the doctor for antibiotics. Antibiotics will help decrease the secretion.”

C

“It is important to increase your activity. A daily brisk walk will help promote drainage.”

D

“Keep a diary if when your symptoms occur. This can help you identify what precipitates your attacks.”

Question 19 Explanation: 

It is important for clients with allergic rhinitis to determine the precipitating factors so that they can be avoided. Keeping a diary can help identify these triggers. Nasal decongestant sprays should not be used regularly because they can cause a rebound effect. Antibiotics are not appropriate. Increasing activity will not control the client’s symptoms; in fact, walking outdoors may increase them if the client is allergic to pollen.

Question 20

A nurse is caring for a client with diabetic ketoacidosis and documents that the client is experiencing Kussmaul’s respirations. Based on this documentation, which of the following did the nurse observe?

A

Respirations that are abnormally deep, regular, and increased in rate.

B

Respirations that are regular but abnormally slow.

C

Respirations that are labored and increased in depth and rate

D

Respirations that cease for several seconds.

Question 20 Explanation: 

Kussmaul’s respirations are abnormally deep, regular, and increased in rate.

Question 21

Theophylline (Theo-Dur) tablets are prescribed for a client with chronic airflow limitation, and the nurse instructs the client about the medication. Which statement by the client indicates a need for further teaching?

A

“I will take the medication on an empty stomach.”

B

“I will take the medication with food.”

C

“I will continue to take the medication even if I am feeling better.”

D

“Periodic blood levels will need to be obtained.”

Question 21 Explanation: 

Theo-Dur is a bronchodilator. The medication should be administered with food such as milk and crackers to prevent GI irritation.

Question 22

Which of the following measures can reduce or prevent the incidence of atelectasis in a post-operative client?

A

Chest physiotherapy

B

Mechanical ventilation

C

Reducing oxygen requirements

D

Use of an incentive spirometer

Question 22 Explanation: 

Using an incentive spirometer requires the client to take deep breaths and promotes lung expansion. Chest physiotherapy helps mobilize secretions but won’t prevent atelectasis. Reducing oxygen requirements or placing someone on mechanical ventilation doesn’t affect the development of atelectasis.

Question 23

Basilar crackles are present in a client’s lungs on auscultation. The nurse knows that these are discrete, noncontinuous sounds that are:

A

Caused by the sudden opening of alveoli

B

Usually more prominent during expiration

C

Produced by airflow across passages narrowed by secretions

D

Found primarily in the pleura.

Question 23 Explanation: 

Basilar crackles are usually heard during inspiration and are caused by sudden opening of the alveoli.

Question 24

If a client continues to hypoventilate, the nurse will continually assess for a complication of:

A

Respiratory acidosis

B

Respiratory alkalosis

C

Metabolic acidosis

D

Metabolic alkalosis

Question 24 Explanation: 

Respiratory acidosis represents an increase in the acid component, carbon dioxide, and an increase in the hydrogen ion concentration (decreased pH) of the arterial blood.

Question 25

A female client comes into the emergency room complaining of SOB and pain in the lung area. She states that she started taking birth control pills 3 weeks ago and that she smokes. Her VS are: 140/80, P 110, R 40. The physician orders ABG’s, results are as follows: pH: 7.50; PaCO2 29 mm Hg; PaO2 60 mm Hg; HCO3– 24 mEq/L; SaO2 86%. Considering these results, the first intervention is to:

A

Begin mechanical ventilation

B

Place the client on oxygen

C

Give the client sodium bicarbonate

D

Monitor for pulmonary embolism.

Question 25 Explanation: 

The pH (7.50) reflects alkalosis, and the low PaCO2 indicated the lungs are involved. The client should immediately be placed on oxygen via mask so that the SaO2 is brought up to 95%. Encourage slow, regular breathing to decrease the amount of CO2 she is losing. This client may have pulmonary embolism, so she should be monitored for this condition (D), but it is not the first intervention. Sodium bicarbonate (C) would be given to reverse acidosis; mechanical ventilation (A) may be ordered for acute respiratory acidosis.

Question 26

A nurse is caring for a client with renal failure. Blood gas results indicate a pH of 7.30; a PCO2 of 32 mm Hg, and a bicarbonate concentration of 20 mEq/L. The nurse has determined that the client is experiencing metabolic acidosis. Which of the following laboratory values would the nurse expect to note?

A

Sodium level of 145 mEq/L

B

Magnesium level of 2.0 mg/dL

C

Potassium level of 5.2 mEq/L

D

Phosphorus level of 4.0 mg/dL

Question 26 Explanation: 

Clinical manifestations of metabolic acidosis include hyperpnea with Kussmaul’s respirations; headache; N/V, and diarrhea; fruity-smelling breath resulting from improper fat metabolism; CNS depression, including mental dullness, drowsiness, stupor, and coma; twitching, and coma. Hyperkalemia will occur.

Question 27

When developing a discharge plan to manage the care of a client with COPD, the nurse should anticipate that the client will do which of the following?

A

Develop infections easily

B

Maintain current status

C

Require less supplemental oxygen

D

Show permanent improvement.

Question 27 Explanation: 

A client with COPD is at high risk for development of respiratory infections. COPD is a slowly progressive; therefore, maintaining current status and establishing a goal that the client will require less supplemental oxygen are unrealistic expectations. Treatment may slow progression of the disease, but permanent improvement is highly unlikely.

Question 28

Emergency treatment for a client with impending anaphylaxis secondary to hypersensitivity to a drug should include which of the following actions first?

A

Administering oxygen

B

Inserting an I.V. catheter

C

Obtaining a complete blood count (CBC)

D

Taking vital signs

Question 28 Explanation: 

Giving oxygen would be the best first action in this case. Vital signs then should be checked and the physician immediately notified. If the client doesn’t already have an I.V. catheter, one may be inserted now if anaphylactic shock is developing. Obtaining a CBC wouldn’t help the emergency situation.

Question 29

Which of the following outcomes would be appropriate for a client with COPD who has been discharged to home? The client:

A

Promises to do pursed lip breathing at home.

B

States actions to reduce pain.

C

States that he will use oxygen via a nasal cannula at 5 L/minute.

D

Agrees to call the physician if dyspnea on exertion increases.

Question 29 Explanation: 

Increasing dyspnea on exertion indicates that the client may be experiencing complications of COPD, and therefore the physician should be notified. Extracting promises from clients is not an outcome criterion. Pain is not a common symptom of COPD. Clients with COPD use low-flow oxygen supplementation (1 to 2 L/minute) to avoid suppressing the respiratory drive, which, for these clients, is stimulated by hypoxia.

Question 30

An oxygenated delivery system is prescribed for a client with COPD to deliver a precise oxygen concentration. Which of the following types of oxygen delivery systems would the nurse anticipate to be prescribed?

A

Venturi mask

B

Aerosol mask

C

Face tent

D

Tracheostomy collar

Question 30 Explanation: 

The venture mask delivers the most accurate oxygen concentration. The Venturi mask is the best oxygen delivery system for the client with chronic airflow limitation because it delivers a precise oxygen concentration. The face tent, the aerosol mask, and the tracheostomy collar are also high-flow oxygen delivery systems but most often are used to administer high humidity.

Question 31

A 66-year-old client has marked dyspnea at rest, is thin, and uses accessory muscles to breathe. He’s tachypneic, with a prolonged expiratory phase. He has no cough. He leans forward with his arms braced on his knees to support his chest and shoulders for breathing. This client has symptoms of which of the following respiratory disorders?

A

ARDS

B

Asthma

C

Chronic obstructive bronchitis

D

Emphysema

Question 31 Explanation: 

These are classic signs and symptoms of a client with emphysema. Clients with ARDS are acutely short of breath and require emergency care; those with asthma are also acutely short of breath during an attack and appear very frightened. Clients with chronic obstructive bronchitis are bloated and cyanotic in appearance.

Question 32

Guaifenesin 300 mg four times daily has been ordered as an expectorant. The dosage strength of the liquid is 200mg/5ml. How many mL should the nurse administer each dose?

A

5.0 ml

B

7.5 ml

C

9.5 ml

D

10 ml

Question 33

A nurse is caring for a client who is on a mechanical ventilator. Blood gas results indicate a pH of 7.50 and a PCO2 of 30 mm Hg. The nurse has determined that the client is experiencing respiratory alkalosis. Which laboratory value would most likely be noted in this condition?

A

Sodium level of 145 mEq/L

B

Potassium level of 3.0 mEq/L

C

Magnesium level of 2.0 mg/L

D

Phosphorus level of 4.0 mg/dl

Question 33 Explanation: 

Clinical manifestations of respiratory alkalosis include headache, tachypnea, paresthesias, tetany, vertigo, convulsions, hypokalemia, and hypocalcemia. Options A, C, and D identify normal laboratory values. Option B identifies the presence of hypokalemia.

Question 34

A client states that the physician said the tidal volume is slightly diminished and asks the nurse what this means. The nurse explains that the tidal volume is the amount of air:

A

Exhaled forcibly after a normal expiration

B

Exhaled after there is a normal inspiration

C

Trapped in the alveoli that cannot be exhaled

D

Forcibly inspired over and above a normal respiration.

Question 34 Explanation: 

Tidal volume (TV) is defined as the amount of air exhaled after a normal inspiration.

Question 35

A 58-year-old client with a 40-year history of smoking one to two packs of cigarettes a day has a chronic cough producing thick sputum, peripheral edema, and cyanotic nail beds. Based on this information, he most likely has which of the following conditions?

A

Adult respiratory distress syndrome (ARDS)

B

Asthma

C

Chronic obstructive bronchitis

D

Emphysema

Question 35 Explanation: 

Because of his extensive smoking history and symptoms, the client most likely has chronic obstructive bronchitis. Clients with ARDS have acute symptoms of and typically need large amounts of oxygen. Clients with asthma and emphysema tend not to have a chronic cough or peripheral edema.

Question 36

The client with asthma should be taught that which of the following is one of the most common precipitating factors of an acute asthma attack?

A

Occupational exposure to toxins

B

Viral respiratory infections

C

Exposure to cigarette smoke

D

Exercising in cold temperatures

Question 36 Explanation: 

The most common precipitator of asthma attacks is viral respiratory infection. Clients with asthma should avoid people who have the flu or a cold and should get yearly flu vaccinations. Environmental exposure to toxins or heavy particulate matter can trigger asthma attacks; however, far fewer asthmatics are exposed to such toxins than are exposed to viruses. Cigarette smoke can also trigger asthma attacks, but to a lesser extent than viral respiratory infections. Some asthmatic attacks are triggered by exercising in cold weather.

Question 37

Which of the following would be an expected outcome for a client recovering from an upper respiratory tract infection? The client will:

A

Maintain a fluid intake of 800ml every 24 hours.

B

Experience chills only once a day

C

Cough productively without chest discomfort.

D

Experience less nasal obstruction and discharge.

Question 37 Explanation: 

A client recovering from an URI should report decreasing or no nasal discharge and obstruction. Daily fluid intake should be increase to more than 1 L every 24 hours to liquefy secretions. The temperature should be below 100*F (37.8*C) with no chills or diaphoresis. A productive cough with chest pain indicated pulmonary infection, not an URI.

Question 38

A fifty-year-old client has a tracheostomy and requires tracheal suctioning. The first intervention in completing this procedure would be to:

A

Change the tracheostomy dressing

B

Provide humidity with a trach mask

C

Apply oral or nasal suction

D

Deflate the tracheal cuff

Question 38 Explanation: 

Before deflating the tracheal cuff (D), the nurse will apply oral or nasal suction to the airway to prevent secretions from falling into the lung. Dressing change (A) and humidity (B) do not relate to suctioning.

Question 39

A cyanotic client with an unknown diagnosis is admitted to the E.R. In relation to oxygen, the first nursing action would be to:

A

Wait until the client’s lab work is done.

B

Not administer oxygen unless ordered by the physician.

C

Administer oxygen at 2 L flow per minute.

D

Administer oxygen at 10 L flow per minute and check the client’s nailbeds.

Question 39 Explanation: 

Administer oxygen at 2 L/minute and no more, for if the client if emphysemic and receives too high a level of oxygen, he will develop CO2 narcosis and the respiratory system will cease to function.

Question 40

A client with acute asthma showing inspiratory and expiratory wheezes and a decreased expiratory volume should be treated with which of the following classes of medication right away?

A

Beta-adrenergic blockers

B

Bronchodilators

C

Inhaled steroids

D

Oral steroids

Question 40 Explanation: 

Bronchodilators are the first line of treatment for asthma because bronchoconstriction is the cause of reduced airflow. Beta-adrenergic blockers aren’t used to treat asthma and can cause bronchoconstriction. Inhaled or oral steroids may be given to reduce the inflammation but aren’t used for emergency relief.

Question 41

Which of the following assessment findings would help confirm a diagnosis of asthma in a client suspected of having the disorder?

A

Circumoral cyanosis

B

Increased forced expiratory volume

C

Inspiratory and expiratory wheezing

D

Normal breath sounds

Question 41 Explanation: 

Inspiratory and expiratory wheezes are typical findings in asthma. Circumoral cyanosis may be present in extreme cases of respiratory distress. The nurse would expect the client to have a decreased forced expiratory volume because asthma is an obstructive pulmonary disease. Breath sounds will be “tight” sounding or markedly decreased; they won’t be normal.

Question 42

A nurse is caring for a client with an ileostomy understands that the client is most at risk for developing which acid-base disorder?

A

Respiratory acidosis

B

Respiratory alkalosis

C

Metabolic acidosis

D

Metabolic alkalosis

Question 42 Explanation: 

Intestinal secretions are high in bicarbonate and may be lost through enteric drainage tubes or an ileostomy or with diarrhea (remember, diarrhea is coming out of thebase). These conditions result in metabolic acidosis.

Question 43

A client with acute asthma is prescribed short-term corticosteroid therapy. What is the rationale for the use of steroids in clients with asthma?

A

Corticosteroids promote bronchodilation

B

Corticosteroids act as an expectorant

C

Corticosteroids have an anti-inflammatory effect

D

Corticosteroids prevent development of respiratory infections.

Question 43 Explanation: 

Corticosteroids have an anti-inflammatory effect and act to decrease edema in the bronchial airways and decrease mucus secretion. Corticosteroids do not have a bronchodilator effect, act as expectorants, or prevent respiratory infections.

Question 44

The most reliable index to determine the respiratory status of a client is to:

A

Observe the chest rising and falling

B

Observe the skin and mucous membrane color.

C

Listen and feel the air movement.

D

Determine the presence of a femoral pulse.

Question 44 Explanation: 

To check for breathing, the nurse places her ear and cheek next to the client’s mouth and nose to listen and feel for air movement. The chest rising and falling (A) is not conclusive of a patent airway. Observing skin color (B) is not an accurate assessment of respiratory status, nor is checking the femoral pulse.

Question 45

Which of the following individuals would the nurse consider to have the highest priority for receiving an influenza vaccination?

A

A 60-year-old man with a hiatal hernia

B

A 36-year-old woman with 3 children

C

A 50-year-old woman caring for a spouse with cancer

D

A 60-year-old woman with osteoarthritis

Question 45 Explanation: 

Individuals who are household members or home care providers for high-risk individuals are high-priority targeted groups for immunization against influenza to prevent transmission to those who have a decreased capacity to deal with the disease. The wife who is caring for a husband with cancer has the highest priority of the clients described.

Question 46

Lisa is newly diagnosed with asthma and is being discharged from the hospital after an episode of status asthmaticus. Discharge teaching should include which of the following:

A

Limitations in sports that will be imposed by the illness

B

Specific instructions on staying cal during an attack

C

The relationship of symptoms and a specific trigger such as physical exercise

D

Incidence of status asthmaticus in children and teens

Question 46 Explanation: 

COPD clients have low oxygen and high carbon dioxide levels. Therefore, hypoxia is the main stimulus for ventilation is persons with chronic hypercapnea. Increasing the level of oxygen would decrease the stimulus to breathe.

Question 47

Which of the following health promotion activities should the nurse include in the discharge teaching plan for a client with asthma?

A

Incorporate physical exercise as tolerated into the treatment plan.

B

Monitor peak flow numbers after meals and at bedtime.

C

Eliminate stressors in the work and home environment

D

Use sedatives to ensure uninterrupted sleep at night.

Question 47 Explanation: 

Physical exercise is beneficial and should be incorporated as tolerated into the client’s schedule. Peak flow numbers should be monitored daily, usually in the morning (before taking medication). Peak flow does not need to be monitored after each meal. Stressors in the client’s life should be modified but cannot be totally eliminated. Although adequate sleep is important, it is not recommended that sedatives be routinely taken to induce sleep.

Question 48

Following the initial care of a client with asthma and impending anaphylaxis from hypersensitivity to a drug, the nurse should take which of the following steps next?

A

Administer beta-adrenergic blockers

B

Administer bronchodilators

C

Obtain serum electrolyte levels

D

Have the client lie flat in the bed.

Question 48 Explanation: 

Bronchodilators would help open the client’s airway and improve his oxygenation status. Beta-adrenergic blockers aren’t indicated in the management of asthma because they may cause bronchospasm. Obtaining laboratory values wouldn’t be done on an emergency basis, and having the client lie flat in bed could worsen his ability to breathe.

Question 49

An acceleration in oxygen dissociation from hemoglobin, and thus oxygen delivery to the tissues, is caused by:

A

A decreasing oxygen pressure in the blood

B

An increasing carbon dioxide pressure in the blood

C

A decreasing oxygen pressure and/or an increasing carbon dioxide pressure in the blood.

D

An increasing oxygen pressure and/or a decreasing carbon dioxide pressure in the blood.

Question 49 Explanation: 

The lower the PO2 and the higher the PCO2, the more rapidly oxygen dissociated from the oxy-hemoglobin molecule.

Question 50

A 19-year-old comes into the emergency department with acute asthma. His respiratory rate is 44 breaths/minute, and he appears to be in acute respiratory distress. Which of the following actions should be taken first?

A

Take a full medication history

B

Give a bronchodilator by neubulizer

C

Apply a cardiac monitor to the client

D

Provide emotional support to the client.

Question 50 Explanation: 

The client is having an acute asthma attack and needs to increase oxygen delivery to the lung and body. Nebulized bronchodilators open airways and increase the amount of oxygen delivered. First resolve the acute phase of the attack ad how to prevent attacks in the future. It may not be necessary to place the client on a cardiac monitor because he’s only 19-years-old, unless he has a past medical history of cardiac problems.

Question 51

A client is prescribed metaproterenol (Alupent) via a metered dose inhaler (MDI), two puffs every 4 hours. The nurse instructs the client to report side effects. Which of the following are potential side effects of metaproterenol?

A

Irregular heartbeat

B

Constipation

C

Petal edema

D

Decreased heart rate.

Question 51 Explanation: 

Irregular heart rates should be reported promptly to the care provider. Metaproterenol may cause irregular heartbeat, tachycardia, or anginal pain because of its adrenergic effect on the beta-adrenergic receptors in the heart. It is not recommended for use in clients with known cardiac disorders. Metaproterenol does not cause constipation, petal edema, or bradycardia.

Question 52

A nurse is caring for a client after a bronchoscopy and biopsy. Which of the following signs if noted in the client should be reported immediately to the physician?    

A

Blood-streaked sputum

B

Dry cough

C

Hematuria

D

Bronchospasm

Question 52 Explanation: 

If a biopsy was performed during a bronchoscopy, blood streaked sputum is expected for several hours. Frank blood indicates hemorrhage. A dry cough may be expected. The client should be assessed for signs of complications, which would include cyanosis, dyspnea, stridor, bronchospasm, hemoptysis, hypotension, tachycardia, and dysrhythmias. Hematuria is unrelated to this procedure.

Question 53

A client is in danger of respiratory arrest following the administration of a narcotic analgesic. An arterial blood gas value is obtained. The nurse would expect to PaCO2 to be which of the following values?

A

15 mm Hg

B

30 mm Hg

C

40 mm Hg

D

80 mm Hg

Question 53 Explanation: 

A client about to go into respiratory arrest will have inefficient ventilation and will be retaining carbon dioxide. The value expected would be around 80 mm Hg. All other values are lower than expected.

Question 54

Emergency treatment of a client in status asthmaticus includes which of the following medications?

A

Inhaled beta-adrenergic agents

B

Inhaled corticosteroids

C

I.V. beta-adrenergic agents

D

Oral corticosteroids

Question 54 Explanation: 

Inhaled beta-adrenergic agents help promote bronchodilation, which improves oxygenation. I.V. beta-adrenergic agents can be used but have to be monitored because of their greater systemic effects. They’re typically used when the inhaled beta-adrenergic agents don’t work. Corticosteriods are slow-acting, so their use won’t reduce hypoxia in the acute phase.

Question 55

A client’s ABG results are as follows: pH: 7.16; PaCO2 80 mm Hg; PaO2 46 mm Hg; HCO3– 24 mEq/L; SaO2 81%. This ABG result represents which of the following conditions?

A

Metabolic acidosis

B

Metabolic alkalosis

C

Respiratory acidosis

D

Respiratory alkalosis

Question 55 Explanation: 

You all should know this. Practice some problems if you got this wrong.

Question 56

A client’s arterial blood gas levels are as follows: pH 7.31; PaO2 80 mm Hg, PaCO2 65 mm Hg; HCO3– 36 mEq/L. Which of the following signs or symptoms would the nurse expect?

A

Cyanosis

B

Flushed skin

C

Irritability

D

Anxiety

Question 56 Explanation: 

The high PaCO2 level causes flushing due to vasodilation. The client also becomes drowsy and lethargic because carbon dioxide has a depressant effect on the CNS. Cyanosis is a late sign of hypoxia. Irritability and anxiety are not common with a PaCO2 level of 65 mm Hg but are associated with hypoxia.

Question 57

The physician has scheduled a client for a left pneumonectomy. The position that will most likely be ordered postoperatively for his is the:

A

Unoperative side or back

B

Operative side or back

C

Back only

D

Back or either side.

Question 57 Explanation: 

Positioning the client on the operative side facilitates the accumulation of serisanguineous fluid. The fluid forms a solid mass, which prevents the remaining lung from being drawn into the space.

Question 58

Which of the following physical assessment findings would the nurse expect to find in a client with advanced COPD?

A

Increased anteroposterior chest diameter

B

Underdeveloped neck muscles

C

Collapsed neck veins

D

Increased chest excursions with respiration

Question 58 Explanation: 

Increased anteroposterior chest diameter is characteristic of advanced COPD. Air is trapped in the overextended alveoli, and the ribs are fixed in an inspiratory position. The result is the typical barrel-chested appearance. Overly developed, not underdeveloped, neck muscles are associated with COPD because of their increased use in the work of breathing. Distended, not collapsed, neck veins are associated with COPD as a symptom of the heart failure that the client may experience secondary to the increased workload on the heart to pump into pulmonary vasculature. Diminished, not increased, chest excursion is associated with COPD.

Question 59

Teaching for a client with chronic obstructive pulmonary disease (COPD) should include which of the following topics?

A

How to have his wife learn to listen to his lungs with a stethoscope from Wal-Mart.

B

How to increase his oxygen therapy.

C

How to treat respiratory infections without going to the physician.

D

How to recognize the signs of an impending respiratory infection.

Question 59 Explanation: 

Respiratory infection in clients with a respiratory disorder can be fatal. It’s important that the client understands how to recognize the signs and symptoms of an impending respiratory infection. It isn’t appropriate for the wife to listen to his lung sounds, besides, you can’t purchase stethoscopes from Wal-Mart. If the client has signs and symptoms of an infection, he should contact his physician at once.

Question 60

An elderly client has been ill with the flu, experiencing headache, fever, and chills. After 3 days, she develops a cough productive of yellow sputum. The nurse auscultates her lungs and hears diffuse crackles. How would the nurse best interpret these assessment findings?

A

It is likely that the client is developing a secondary bacterial pneumonia.

B

The assessment findings are consistent with influenza and are to be expected.

C

The client is getting dehydrated and needs to increase her fluid intake to decrease secretions.

D

The client has not been taking her decongestants and bronchodilators as prescribed.

Question 60 Explanation: 

Pneumonia is the most common complication of influenza, especially in the elderly. The development of a purulent cough and crackles may be indicative of a bacterial infection are not consistent with a diagnosis of influenza. These findings are not indicative of dehydration. Decongestants and bronchodilators are not typically prescribed for the flu.

Question 61

A 7-year-old client is brought to the E.R. He’s tachypneic and afebrile and has a respiratory rate of 36 breaths/minute and a nonproductive cough. He recently had a cold. From his history, the client may have which of the following?

A

Acute asthma

B

Bronchial pneumonia

C

Chronic obstructive pulmonary disease (COPD)

D

Emphysema

Question 61 Explanation: 

Based on the client’s history and symptoms, acute asthma is the most likely diagnosis. He’s unlikely to have bronchial pneumonia without a productive cough and fever and he’s too young to have developed COPD or emphysema.

Question 62

The nurse would anticipate which of the following ABG results in a client experiencing a prolonged, severe asthma attack?

A

Decreased PaCO2, increased PaO2, and decreased pH.

B

Increased PaCO2, decreased PaO2, and decreased pH.

C

Increased PaCO2, increased PaO2, and increased pH.

D

Decreased PaCO2, decreased PaO2, and increased pH.

Question 62 Explanation: 

As the severe asthma attack worsens, the client becomes fatigued and alveolar hypotension develops. This leads to carbon dioxide retention and hypoxemia. The client develops respiratory acidosis. Therefore, the PaCO2 level increase, the PaO2 level decreases, and the pH decreases, indicating acidosis.

Question 63

Which of the following ABG abnormalities should the nurse anticipate in a client with advanced COPD?

A

Increased PaCO2

B

Increased PaO2

C

Increased pH.

D

Increased oxygen saturation

Question 63 Explanation: 

As COPD progresses, the client typically develops increased PaCO2 levels and decreased PaO2 levels. This results in decreased pH and decreased oxygen saturation. These changes are the result of air trapping and hypoventilation.

Question 64

An unconscious client is admitted to an emergency room. Arterial blood gas measurements reveal a pH of 7.30, a low bicarbonate level, a normal carbon dioxide level, and a normal oxygen level. An elevated potassium level is also present. These results indicate the presence of:

A

Metabolic acidosis

B

Respiratory acidosis

C

Combined respiratory and metabolic acidosis

D

overcompensated respiratory acidosis

Question 64 Explanation: 

In an acidotic condition the pH would be low, indicating the acidosis. In addition, a low bicarbonate level along with the pH would indicate a metabolic state.

Question 65

The nurse is planning to teach a client with COPD how to cough effectively. Which of the following instructions should be included?

A

Take a deep abdominal breath, bend forward, and cough 3 to 4 times on exhalation.

B

Lie flat on back, splint the thorax, take two deep breaths and cough.

C

Take several rapid, shallow breaths and then cough forcefully.

D

Assume a side-lying position, extend the arm over the head, and alternate deep breathing with coughing.

Question 65 Explanation: 

The goal of effective coughing is to conserve energy, facilitate removal of secretions, and minimize airway collapse. The client should assume a sitting position with feet on the floor if possible. The client should bend forward slightly and, using pursed-lip breathing, exhale. After resuming an upright position, the client should use abdominal breathing to slowly and deeply inhale. After repeating this process 3 or 4 times, the client should take a deep abdominal breath, bend forward and cough 3 or 4 times upon exhalation (“huff” cough). Lying flat does not enhance lung expansion; sitting upright promotes full expansion of the thorax. Shallow breathing does not facilitate removal of secretions, and forceful coughing promotes collapse of airways. A side-lying position does not allow for adequate chest expansion to promote deep breathing.

Question 66

A client with COPD reports steady weight loss and being “too tired from just breathing to eat.” Which of the following nursing diagnoses would be most appropriate when planning nutritional interventions for this client?

A

Altered nutrition: Less than body requirements related to fatigue.

B

Activity intolerance related to dyspnea.

C

Weight loss related to COPD.

D

Ineffective breathing pattern related to alveolar hypoventilation.

Question 66 Explanation: 

The client’s problem is altered nutrition—specifically, less than required. The cause, as stated by the client, is the fatigue associated with the disease process. Activity intolerance is a likely diagnosis but is not related to the client’s nutritional problems. Weight loss is not a nursing diagnosis. Ineffective breathing pattern may be a problem, but this diagnosis does not specifically address the problem of weight loss described by the client.

Question 67

A nurse is assessing a client with chronic airflow limitation and notes that the client has a “barrel chest.” The nurse interprets that this client has which of the following forms of chronic airflow limitation?

A

Chronic obstructive bronchitis

B

Emphysema

C

Bronchial asthma

D

Bronchial asthma and bronchitis

Question 67 Explanation: 

The client with emphysema has hyperinflation of the alveoli and flattening of the diaphragm. These lead to increased anteroposterior diameter, which is referred to as “barrel chest.” The client also has dyspnea with prolonged expiration and has hyperresonant lungs to percussion.

Question 68

Pseudoephedrine (Sudafed) has been ordered as a nasal decongestant. Which of the following is a possible side effect of this drug?

A

Constipation

B

Bradycardia

C

Diplopia

D

Restlessness

Question 68 Explanation: 

Side effects of pseudoephedrine are experienced primarily in the cardiovascular system and through sympathetic effects on the CNS. The most common CNS effects include restlessness, dizziness, tension, anxiety, insomnia, and weakness. Common cardiovascular side effects include tachycardia, hypertension, palpitations, and arrhythmias. Constipation and diplopia are not side effects of pesudoephedrine. Tachycardia, not bradycardia, is a side effect of pseudoephedrine.

Question 69

A client is scheduled for blood to be drawn from the radial artery for an ABG determination. Before the blood is drawn, an Allen’s test is performed to determine the adequacy of the:

A

Popliteal circulation

B

Ulnar circulation

C

Femoral circulation

D

Carotid circulation

Question 69 Explanation: 

Before radial puncture for obtaining an ABG, you should perform an Allen’s test to determine adequate ulnar circulation. Failure to determine the presence of adequate collateral circulation could result in severe ischemic injury o the hand if damage to the radial artery occurs with arterial puncture.

Question 70

The term “pink puffer” refers to the client with which of the following conditions?

A

ARDS

B

Asthma

C

Chronic obstructive bronchitis

D

Emphysema

Question 70 Explanation: 

Because of the large amount of energy it takes to breathe, clients with emphysema are usually cachectic. They’re pink and usually breathe through pursed lips, hence the term “puffer”. Clients with ARDS are usually acutely short of breath. Clients with asthma don’t have any particular characteristics, and clients with chronic obstructive bronchitis are bloated and cyanotic in appearance.

Question 71

A nurse understands that the excessive use of oral antacids containing bicarbonate can result in which acid-base disturbance?

A

Respiratory alkalosis

B

Respiratory acidosis

C

Metabolic acidosis

D

Metabolic alkalosis

Question 71 Explanation: 

Increases in base components occur as a result of oral or parenteral intake of bicarbonates, carbonates, acetates, citrates, or lactates. Excessive use of oral antacids containing bicarbonate can cause metabolic alkalosis.

Question 72

A nurse reviews the ABG values and notes a pH of 7.50, a PCO2 of 30 mm Hg, and an HCO3 of 25 mEq/L. The nurse interprets these values as indicating:

A

Respiratory acidosis uncompensated

B

Respiratory alkalosis uncompensated

C

Metabolic acidosis uncompensated

D

Metabolic acidosis partially compensated.

Question 72 Explanation: 

In respiratory alkalosis the pH will be higher than normal and the PCO2 will be low.

Question 73

Assessing a client who has developed atelectasis postoperatively, the nurse will most likely find:

A

A flushed face

B

Dyspnea and pain

C

Decreased temperature

D

Severe cough and no pain.

Question 73 Explanation: 

Atelectasis is a collapse of the alveoli due to obstruction or hypoventilation. Clients become short of breath, have a high temperature, and usually experience severe pain but do not have a severe cough (D). The shortness of breath is a result of decreased oxygen-carbon dioxide exchange at the alveolar level.

Question 74

Which of the following additional assessment data should immediately be gathered to determine the status of a client with a respiratory rate of 4 breaths/minute?

A

Arterial blood gas (ABG) and breath sounds

B

Level of consciousness and a pulse oximetry value.

C

Breath sounds and reflexes

D

Pulse oximetry value and heart sounds

Question 74 Explanation: 

First, the nurse should attempt to rouse the client because this should increase the client’s respiratory rate. If available, a spot pulse oximetry check should be done and breath sounds should be checked. The physician should be notified immediately if of the findings. He’ll probably order ABG analysis to determine specific carbon dioxide and oxygen levels, which will indicate the effectiveness of ventilation. Reflexes and heart sounds will be part of the more extensive examination done after these initial actions are completed.

Question 75

Which of the following is a priority goal for the client with COPD?

A

Maintaining functional ability

B

Minimizing chest pain

C

Increasing carbon dioxide levels in the blood

D

Treating infectious agents

Question 75 Explanation: 

A priority goal for the client with COPD is to manage the s/s of the disease process so as to maintain the client’s functional ability. Chest pain is not a typical sign of COPD. The carbon dioxide concentration in the blood is increased to an abnormal level in clients with COPD; it would not be a goal to increase the level further. Preventing infection would be a goal of care for the client with COPD.

Question 76

Which of the following pathophysiological mechanisms that occurs in the lung parenchyma allows pneumonia to develop?

A

Atelectasis

B

Bronchiectasis

C

Effusion

D

Inflammation

Question 76 Explanation: 

The most common feature of all types of pneumonia is an inflammatory pulmonary response to the offending organism or agent. Atelectasis and brochiectasis indicate a collapse of a portion of the airway that doesn’t occur with pneumonia. An effusion is an accumulation of excess pleural fluid in the pleural space, which may be a secondary response to pneumonia.

Question 77

A nurse is preparing to obtain a sputum specimen from a client. Which of the following nursing actions will facilitate obtaining the specimen?

A

Limiting fluids

B

Having the client take 3 deep breaths.

C

Asking the client to spit into the collection container.

D

Asking the client to obtain the specimen after eating.

Question 77 Explanation: 

To obtain a sputum specimen, the client should rinse the mouth to prevent contamination, breathe deeply, and then cough unto a sputum specimen container. The client should be encouraged to cough and not spit so as to obtain sputum. Sputum can be thinned by fluids or by a respiratory treatment such as inhalation of nebulized saline or water. The optimal time to obtain a specimen is on arising in the morning.

Question 78

Aminophylline (theophylline) is prescribed for a client with acute bronchitis. A nurse administers the medication, knowing that the primary action of this medication is to:

A

Promote expectoration

B

Suppress the cough

C

Relax smooth muscles of the bronchial airway

D

Prevent infection

Question 78 Explanation: 

Aminophylline is a bronchodilator that directly relaxes the smooth muscles of the bronchial airway.

Question 79

A client with emphysema should receive only 1 to 3 L/minute of oxygen, if needed, or he may lose his hypoxic drive. Which of the following statements is correct about hypoxic drive?

A

The client doesn’t notice he needs to breathe.

B

The client breathes only when his oxygen levels climb above a certain point.

C

The client breathes only when his oxygen levels dip below a certain point.

D

The client breathes only when his carbon dioxide level dips below a certain point.

Question 79 Explanation: 

Clients with emphysema breathe when their oxygen levels drop to a certain level; this is known as the hypoxic drive. They don’t take a breath when their levels of carbon dioxide are higher than normal, as do those with healthy respiratory physiology. If too much oxygen is given, the client has little stimulus to take another breath. In the meantime, his carbon dioxide levels continue to climb, and the client will pass out, leading to a respiratory arrest.

Question 80

Exercise has which of the following effects on clients with asthma, chronic bronchitis, and emphysema?

A

It enhances cardiovascular fitness.

B

It improves respiratory muscle strength.

C

It reduces the number of acute attacks.

D

It worsens respiratory function and is discouraged.

Question 80 Explanation: 

Exercise can improve cardiovascular fitness and help the client tolerate periods of hypoxia better, perhaps reducing the risk of heart attack. Most exercise has little effect on respiratory muscle strength, and these clients can’t tolerate the type of exercise necessary to do this. Exercise won’t reduce the number of acute attacks. In some instances, exercise may be contraindicated, and the client should check with his physician before starting any exercise program.

Question 81

The nurse teaches a client with COPD to assess for s/s of right-sided heart failure. Which of the following s/s would be included in the teaching plan?

A

Clubbing of nail beds

B

Hypertension

C

Peripheral edema

D

Increased appetite

Question 81 Explanation: 

Right-sided heart failure is a complication of COPD that occurs because of pulmonary hypertension. Signs and symptoms of right-sided heart failure include peripheral edema, jugular venous distention, hepatomegaly, and weight gain due to increased fluid volume. Clubbing of nail beds is associated with conditions of chronic hypoxia. Hypertension is associated with left-sided heart failure. Clients with heart failure have decreased appetites.

Question 82

An elderly client with pneumonia may appear with which of the following symptoms first?

A

Altered mental status and dehydration

B

fever and chills

C

Hemoptysis and dyspnea

D

Pleuretic chest pain and cough

Question 82 Explanation: 

Fever, chills, hemoptysis, dyspnea, cough, and pleuric chest pain are the common symptoms of pneumonia, but elderly clients may first appear with only an altered mental status and dehydration due to a blunted immune response.

Question 83

A female client is scheduled to have a chest radiograph. Which of the following questions is of most importance to the nurse assessing this client?

A

“Is there any possibility that you could be pregnant?”

B

“Are you wearing any metal chains or jewelry?”

C

“Can you hold your breath easily?”

D

“Are you able to hold your arms above your head?”

Question 83 Explanation: 

The most important item to ask about is the client’s pregnancy status because pregnant women should not be exposed to radiation. Clients are also asked to remove any chains or metal objects that could interfere with obtaining an adequate film. A chest radiograph most often is done at full inspiration, which gives optimal lung expansion. If a lateral view of the chest is ordered, the client is asked to raise the arms above the head. Most films are done in posterior-anterior view.

Question 84

Which of the following treatment goals is best for the client with status asthmaticus?

A

Avoiding intubation

B

Determining the cause of the attack

C

Improving exercise tolerance

D

Reducing secretions

Question 84 Explanation: 

Inhaled beta-adrenergic agents, I.V. corticosteroids, and supplemental oxygen are used to reduce bronchospasm, improve oxygenation, and avoid intubation. Determining the trigger for the client’s attack and improving exercise tolerance are later goals. Typically, secretions aren’t a problem in status asthmaticus.

Question 85

Which of the following is the primary reason to teach pursed-lip breathing to clients with emphysema?

A

To promote oxygen intake

B

To strengthen the diaphragm

C

To strengthen the intercostal muscles

D

To promote carbon dioxide elimination

Question 85 Explanation: 

Pursed lip breathing prolongs exhalation and prevents air trapping in the alveoli, thereby promoting carbon dioxide elimination. By prolonged exhalation and helping the client relax, pursed-lip breathing helps the client learn to control the rate and depth of respiration. Pursed-lip breathing does not promote the intake of oxygen, strengthen the diaphragm, or strengthen intercostal muscles.

Question 86

Dani was given dilaudid for pain. She’s sleeping and her respiratory rate is 4 breaths/minute. If action isn’t taken quickly, she might have which of the following reactions?

A

Asthma attack

B

Respiratory arrest

C

Be pissed about receiving Narcan

D

Wake up on her own

Question 86 Explanation: 

Narcotics can cause respiratory arrest if given in large quantities. Its unlikely Dani will have an asthma attack or wake up on her own. She may be pissed for a minute, but then she’d be grateful for saving her butt.

Question 87

Clients with chronic obstructive bronchitis are given diuretic therapy. Which of the following reasons best explains why?

A

Reducing fluid volume reduces oxygen demand.

B

Reducing fluid volume improves clients’ mobility.

C

Restricting fluid volume reduces sputum production.

D

Reducing fluid volume improves respiratory function.

Question 87 Explanation: 

Reducing fluid volume reduces the workload of the heart, which reduces oxygen demand and, in turn, reduces the respiratory rate. It may also reduce edema and improve mobility a little, but exercise tolerance will still be harder to clear airways. Reducing fluid volume won’t improve respiratory function, but may improve oxygenation.

Question 88

A nurse is suctioning fluids from a client via a tracheostomy tube. When suctioning, the nurse must limit the suctioning to a maximum of:

A

5 seconds

B

10 seconds

C

30 seconds

D

1 minute

Question 88 Explanation: 

Hypoxemia can be caused by prolonged suctioning, which stimulates the pacemaker cells within the heart. A vasovagal response may occurm causing bradycardia. The nurse must preoxygenate the client before suctioning and limit the suctioning pass to 10 seconds.

Question 89

A 69-year-old client appears thin and cachectic. He’s short of breath at rest and his dyspnea increases with the slightest exertion. His breath sounds are diminished even with deep inspiration. These signs and symptoms fit which of the following conditions?

A

ARDS

B

Asthma

C

Chronic obstructive bronchitis

D

Emphysema

Question 89 Explanation: 

In emphysema, the wall integrity of the individual air sacs is damaged, reducing the surface area available for gas exchange. Very little air movement occurs in the lungs because of bronchiole collapse, as well. In ARDS, the client’s condition is more acute and typically requires mechanical ventilation. In asthma and bronchitis, wheezing is prevalent.

Question 90

A nurse is preparing to obtain an arterial blood gas specimen from a client and plans to perform the Allen’s test on the client. Number in order of priority the steps for performing the Allen’s test .

  1. Ask the client to open and close the hand repeatedly.
  2. Apply pressure over the ulnar and radial arteries.
  3. Assess the color of the extremity distal to the pressure point
  4. Release pressure from the ulnar artery
  5. Explain the procedure to the client.

A

E, B, A, D, and then C.

B

E, A, B, D, and then C.

C

E, B, A, C, and then D.

D

E, B, C, D, and then A.

Question 91

A nurse reviews the arterial blood gas results of a patient and notes the following: pH 7.45; PCO2 30 mm Hg; and bicarbonate concentration of 22 mEq/L. The nurse analyzes these results as indicating:

A

Metabolic acidosis, compensated.

B

Metabolic alkalosis, uncompensated.

C

Respiratory alkalosis, compensated.

D

Respiratory acidosis, compensated.

Question 91 Explanation: 

The normal pH is 7.35 to 7.45. In a respiratory condition, an opposite (see-saw) will be seen between the pH and the PCO2. In this situation, the pH is at the high end of the normal value and the PCO2 is low. In an alkalotic condition, the pH is up. Therefore, the values identified in the question indicate a respiratory alkalosis. Compensation occurs when the pH returns to a normal value. Because the pH is in the normal range at the high end, compensation has occurred.

Question 92

A nurse is caring for a client with a nasogastric tube that is attached to low suction. The nurse monitors the client, knowing that the client is at risk for which acid-base disorder?

A

Respiratory acidosis

B

Respiratory alkalosis

C

Metabolic acidosis

D

Metabolic alkalosis

Question 92 Explanation: 

Loss of gastric fluid via nasogastric suction or vomiting causes metabolic alkalosis as a result of the loss of hydrochloric acid.

Question 93

A 34-year-old woman with a history of asthma is admitted to the emergency department. The nurse notes that the client is dyspneic, with a respiratory rate of 35 breaths/minute, nasal flaring, and use of accessory muscles. Auscultation of the lung fields reveals greatly diminished breath sounds. Based on these findings, what action should the nurse take to initiate care of the client?

A

Initiate oxygen therapy and reassess the client in 10 minutes.

B

Draw blood for an ABG analysis and send the client for a chest x-ray.

C

Encourage the client to relax and breathe slowly through the mouth

D

Administer bronchodilators

Question 93 Explanation: 

In an acute asthma attack, diminished or absent breath sounds can be an ominous sign of indicating lack of air movement in the lungs and impending respiratory failure. The client requires immediate intervention with inhaled bronchodilators, intravenous corticosteroids, and possibly intravenous theophylline. Administering oxygen and reassessing the client 10 minutes later would delay needed medical intervention, as would drawing an ABG and obtaining a chest x-ray. It would be futile to encourage the client to relax and breathe slowly without providing necessary pharmacologic intervention.

Question 94

A client is admitted to the hospital with acute bronchitis. While taking the client’s VS, the nurse notices he has an irregular pulse. The nurse understands that cardiac arrhythmias in chronic respiratory distress are usually the result of:

A

Respiratory acidosis

B

A build-up of carbon dioxide

C

A build-up of oxygen without adequate expelling of carbon dioxide.

D

An acute respiratory infection.

Question 94 Explanation: 

The arrhythmias are caused by a build-up of carbon dioxide and not enough oxygen so that the heart is in a constant state of hypoxia.

Question 95

A client is receiving isoetharine hydrochloride (Bronkosol) via a nebulizer. The nurse monitors the client for which side effect of this medication?

A

Constipation

B

Diarrhea

C

Bradycardia

D

Tachycardia

Question 95 Explanation: 

Side effects that can occur from a beta 2 agonist include tremors, nausea, nervousness, palpitations, tachycardia, peripheral vasodilation, and dryness of the mouth or throat.

Question 96

A client has just returned to a nursing unit following bronchoscopy. A nurse would implement which of the following nursing interventions for this client?

A

Encouraging additional fluids for the next 24 hours

B

Ensuring the return of the gag reflex before offering foods or fluids

C

Administering atropine intravenously

D

Administering small doses of midazolam (Versed).

Question 96 Explanation: 

After bronchoscopy, the nurse keeps the client on NPO status until the gag reflex returns because the preoperative sedation and the local anesthesia impair swallowing and the protective laryngeal reflexes for a number of hours. Additional fluids is unnecessary because no contrast dye is used that would need to be flushed from the system. Atropine and Versed would be administered before the procedure, not after.

Question 97

A client has started a new drug for hypertension. Thirty minutes after he takes the drug, he develops chest tightness and becomes short of breath and tachypneic. He has a decreased level of consciousness. These signs indicate which of the following conditions?

A

Asthma attack

B

Pulmonary embolism

C

respiratory failure

D

Rheumatoid arthritis

Question 97 Explanation: 

The client was reacting to the drug with respiratory signs of impending anaphylaxis, which could lead to eventual respiratory failure. Although the signs are also related to an asthma attack or a pulmonary embolism, consider the new drug first. Rheumatoid arthritis doesn’t manifest these signs.

Question 98

A client with COPD has developed secondary polycythemia. Which nursing diagnosis would be included in the plan of care because of the polycythemia?

A

Fluid volume deficit related to blood loss.

B

Impaired tissue perfusion related to thrombosis

C

Activity intolerance related to dyspnea

D

Risk for infection related to suppressed immune response.

Question 98 Explanation: 

Chronic hypoxia associated with COPD may stimulate excessive RBC production (polycythemia). This results in increased blood viscosity and the risk of thrombosis. The other nursing diagnoses are not applicable in this situation.

Question 99

A client has been taking flunisolide (Aerobid), two inhalations a day, for treatment of asthma. He tells the nurse that he has painful, white patches in his mouth. Which response by the nurse would be the most appropriate?

A

“This is an anticipated side-effect of your medication. It should go away in a couple of weeks.”

B

“You are using your inhaler too much and it has irritated your mouth.”

C

“You have developed a fungal infection from your medication. It will need to be treated with an antibiotic.”

D

“Be sure to brush your teeth and floss daily. Good oral hygiene will treat this problem.”

Question 99 Explanation: 

Use of oral inhalant corticosteroids, such as flunisolide, can lead to the development of oral thrush, a fungal infection. Once developed, thrush must be treated by antibiotic therapy; it will not resolve on its own. Fungal infections can develop even without overuse of the Corticosteroid inhaler. Although good oral hygiene can help prevent development of a fungal infection, it cannot be used alone to treat the problem.

Question 100

A nurse instructs a client to use the pursed lip method of breathing. The client asks the nurse about the purpose of this type of breathing. The nurse responds, knowing that the primary purpose of pursed lip breathing is:

A

Promote oxygen intake

B

Strengthen the diaphragm

C

Strengthen the intercostal muscles

D

Promote carbon dioxide elimination

Question 100 Explanation: 

Pursed lip breathing facilitates maximum expiration for clients with obstructive lung disease. This type of breathing allows better expiration by increasing airway pressure that keeps air passages open during exhalation.

Once you are finished, click the button below. Any items you have not completed will be marked incorrect. Get Results

There are 100 questions to complete.

List

Return

Shaded items are complete.

123456789101112131415161718192021222324252627282930313233343536373839404142434445464748495051525354555657585960616263646566676869707172737475767778798081828384858687888990919293949596979899100End

Return

You have completed

questions

question

Your score is

Correct

Wrong

Partial-Credit

You have not finished your quiz. If you leave this page, your progress will be lost.

Correct Answer

You Selected

Not Attempted

Final Score on Quiz

Attempted Questions Correct

Attempted Questions Wrong

Questions Not Attempted

Total Questions on Quiz

Question Details

Results

Date

Score

Hint

Time allowed

minutes

seconds

Time used

Answer Choice(s) Selected

Question Text

All done

Need more practice!

Keep trying!

Not bad!

Good work!

Perfect!

Exam Mode

Exam Mode – Questions and choices are randomly arranged, time limit of 1min per question, answers and grade will be revealed after finishing the exam.

MSN Exam for Asthma and COPD (EM)*

Please wait while the activity loads.
If this activity does not load, try refreshing your browser. Also, this page requires javascript. Please visit using a browser with javascript enabled.

If loading fails, click here to try again

Choose the letter of the correct answer. You have 100 mins to finish this exam. Good luck!

Start

Congratulations - you have completed MSN Exam for Asthma and COPD (EM)*. You scored %%SCORE%% out of %%TOTAL%%. Your performance has been rated as %%RATING%%

Your answers are highlighted below.

Question 1

Which of the following would be an expected outcome for a client recovering from an upper respiratory tract infection? The client will:

A

Maintain a fluid intake of 800ml every 24 hours.

B

Experience chills only once a day

C

Cough productively without chest discomfort.

D

Experience less nasal obstruction and discharge.

Question 1 Explanation: 

A client recovering from an URI should report decreasing or no nasal discharge and obstruction. Daily fluid intake should be increase to more than 1 L every 24 hours to liquefy secretions. The temperature should be below 100*F (37.8*C) with no chills or diaphoresis. A productive cough with chest pain indicated pulmonary infection, not an URI.

Question 2

A client with acute asthma showing inspiratory and expiratory wheezes and a decreased expiratory volume should be treated with which of the following classes of medication right away?

A

Beta-adrenergic blockers

B

Bronchodilators

C

Inhaled steroids

D

Oral steroids

Question 2 Explanation: 

Bronchodilators are the first line of treatment for asthma because bronchoconstriction is the cause of reduced airflow. Beta-adrenergic blockers aren’t used to treat asthma and can cause bronchoconstriction. Inhaled or oral steroids may be given to reduce the inflammation but aren’t used for emergency relief.

Question 3

A female client comes into the emergency room complaining of SOB and pain in the lung area. She states that she started taking birth control pills 3 weeks ago and that she smokes. Her VS are: 140/80, P 110, R 40. The physician orders ABG’s, results are as follows: pH: 7.50; PaCO2 29 mm Hg; PaO2 60 mm Hg; HCO3– 24 mEq/L; SaO2 86%. Considering these results, the first intervention is to:

A

Begin mechanical ventilation

B

Place the client on oxygen

C

Give the client sodium bicarbonate

D

Monitor for pulmonary embolism.

Question 3 Explanation: 

The pH (7.50) reflects alkalosis, and the low PaCO2 indicated the lungs are involved. The client should immediately be placed on oxygen via mask so that the SaO2 is brought up to 95%. Encourage slow, regular breathing to decrease the amount of CO2 she is losing. This client may have pulmonary embolism, so she should be monitored for this condition (D), but it is not the first intervention. Sodium bicarbonate (C) would be given to reverse acidosis; mechanical ventilation (A) may be ordered for acute respiratory acidosis.

Question 4

The nurse would anticipate which of the following ABG results in a client experiencing a prolonged, severe asthma attack?

A

Decreased PaCO2, increased PaO2, and decreased pH.

B

Increased PaCO2, decreased PaO2, and decreased pH.

C

Increased PaCO2, increased PaO2, and increased pH.

D

Decreased PaCO2, decreased PaO2, and increased pH.

Question 4 Explanation: 

As the severe asthma attack worsens, the client becomes fatigued and alveolar hypotension develops. This leads to carbon dioxide retention and hypoxemia. The client develops respiratory acidosis. Therefore, the PaCO2 level increase, the PaO2 level decreases, and the pH decreases, indicating acidosis.

Question 5

An unconscious client is admitted to an emergency room. Arterial blood gas measurements reveal a pH of 7.30, a low bicarbonate level, a normal carbon dioxide level, and a normal oxygen level. An elevated potassium level is also present. These results indicate the presence of:

A

Metabolic acidosis

B

Respiratory acidosis

C

Combined respiratory and metabolic acidosis

D

overcompensated respiratory acidosis

Question 5 Explanation: 

In an acidotic condition the pH would be low, indicating the acidosis. In addition, a low bicarbonate level along with the pH would indicate a metabolic state.

Question 6

A nurse is preparing to obtain an arterial blood gas specimen from a client and plans to perform the Allen’s test on the client. Number in order of priority the steps for performing the Allen’s test .

  1. Ask the client to open and close the hand repeatedly.
  2. Apply pressure over the ulnar and radial arteries.
  3. Assess the color of the extremity distal to the pressure point
  4. Release pressure from the ulnar artery
  5. Explain the procedure to the client.

A

E, B, A, D, and then C.

B

E, A, B, D, and then C.

C

E, B, A, C, and then D.

D

E, B, C, D, and then A.

Question 7

Which of the following physical assessment findings would the nurse expect to find in a client with advanced COPD?

A

Increased anteroposterior chest diameter

B

Underdeveloped neck muscles

C

Collapsed neck veins

D

Increased chest excursions with respiration

Question 7 Explanation: 

Increased anteroposterior chest diameter is characteristic of advanced COPD. Air is trapped in the overextended alveoli, and the ribs are fixed in an inspiratory position. The result is the typical barrel-chested appearance. Overly developed, not underdeveloped, neck muscles are associated with COPD because of their increased use in the work of breathing. Distended, not collapsed, neck veins are associated with COPD as a symptom of the heart failure that the client may experience secondary to the increased workload on the heart to pump into pulmonary vasculature. Diminished, not increased, chest excursion is associated with COPD.

Question 8

A client is admitted to the hospital with acute bronchitis. While taking the client’s VS, the nurse notices he has an irregular pulse. The nurse understands that cardiac arrhythmias in chronic respiratory distress are usually the result of:

A

Respiratory acidosis

B

A build-up of carbon dioxide

C

A build-up of oxygen without adequate expelling of carbon dioxide.

D

An acute respiratory infection.

Question 8 Explanation: 

The arrhythmias are caused by a build-up of carbon dioxide and not enough oxygen so that the heart is in a constant state of hypoxia.

Question 9

Which of the following ABG abnormalities should the nurse anticipate in a client with advanced COPD?

A

Increased PaCO2

B

Increased PaO2

C

Increased pH.

D

Increased oxygen saturation

Question 9 Explanation: 

As COPD progresses, the client typically develops increased PaCO2 levels and decreased PaO2 levels. This results in decreased pH and decreased oxygen saturation. These changes are the result of air trapping and hypoventilation.

Question 10

A client has an order to have radial ABG drawn. Before drawing the sample, a nurse occludes the:

A

Brachial and radial arteries, and then releases them and observes the circulation of the hand.

B

Radial and ulnar arteries, releases one, evaluates the color of the hand, and repeats the process with the other artery.

C

Radial artery and observes for color changes in the affected hand.

D

Ulnar artery and observes for color changes in the affected hand.

Question 10 Explanation: 

Before drawing an ABG, the nurse assesses the collateral circulation to the hand with Allen’s test. This involves compressing the radial and ulnar arteries and asking the client to close and open the fist. This should cause the hand to become pale. The nurse then releases pressure on one artery and observes whether circulation is restored quickly. The nurse repeats the process, releasing the other artery. The blood sample may be taken safely if collateral circulation is adequate.

Question 11

Which of the following diets would be most appropriate for a client with COPD?

A

Low fat, low cholesterol

B

Bland, soft diet

C

Low-Sodium diet

D

High calorie, high-protein diet

Question 11 Explanation: 

The client should eat high-calorie, high-protein meals to maintain nutritional status and prevent weight loss that results from the increased work of breathing. The client should be encouraged to eat small, frequent meals. A low-fat, low-cholesterol diet is indicated for clients with coronary artery disease. The client with COPD does not necessarily need to follow a sodium-restricted diet, unless otherwise medically indicated.

Question 12

An elderly client has been ill with the flu, experiencing headache, fever, and chills. After 3 days, she develops a cough productive of yellow sputum. The nurse auscultates her lungs and hears diffuse crackles. How would the nurse best interpret these assessment findings?

A

It is likely that the client is developing a secondary bacterial pneumonia.

B

The assessment findings are consistent with influenza and are to be expected.

C

The client is getting dehydrated and needs to increase her fluid intake to decrease secretions.

D

The client has not been taking her decongestants and bronchodilators as prescribed.

Question 12 Explanation: 

Pneumonia is the most common complication of influenza, especially in the elderly. The development of a purulent cough and crackles may be indicative of a bacterial infection are not consistent with a diagnosis of influenza. These findings are not indicative of dehydration. Decongestants and bronchodilators are not typically prescribed for the flu.

Question 13

Immediately following a thoeacentesis, which clinical manifestations indicate that a complication has occurred and the physician should be notified?

A

Serosanguineous drainage from the puncture site

B

Increased temperature and blood pressure

C

Increased pulse and pallor

D

Hypotension and hypothermia

Question 13 Explanation: 

Increased pulse and pallor are symptoms associated with shock. A compromised venous return may occur if there is a mediastinal shift as a result of excessive fluid removal. Usually no more than 1 L of fluid is removed at one time to prevent this from occurring.

Question 14

A nurse is caring for a client after a bronchoscopy and biopsy. Which of the following signs if noted in the client should be reported immediately to the physician?    

A

Blood-streaked sputum

B

Dry cough

C

Hematuria

D

Bronchospasm

Question 14 Explanation: 

If a biopsy was performed during a bronchoscopy, blood streaked sputum is expected for several hours. Frank blood indicates hemorrhage. A dry cough may be expected. The client should be assessed for signs of complications, which would include cyanosis, dyspnea, stridor, bronchospasm, hemoptysis, hypotension, tachycardia, and dysrhythmias. Hematuria is unrelated to this procedure.

Question 15

Which of the following outcomes would be appropriate for a client with COPD who has been discharged to home? The client:

A

Promises to do pursed lip breathing at home.

B

States actions to reduce pain.

C

States that he will use oxygen via a nasal cannula at 5 L/minute.

D

Agrees to call the physician if dyspnea on exertion increases.

Question 15 Explanation: 

Increasing dyspnea on exertion indicates that the client may be experiencing complications of COPD, and therefore the physician should be notified. Extracting promises from clients is not an outcome criterion. Pain is not a common symptom of COPD. Clients with COPD use low-flow oxygen supplementation (1 to 2 L/minute) to avoid suppressing the respiratory drive, which, for these clients, is stimulated by hypoxia.

Question 16

A fifty-year-old client has a tracheostomy and requires tracheal suctioning. The first intervention in completing this procedure would be to:

A

Change the tracheostomy dressing

B

Provide humidity with a trach mask

C

Apply oral or nasal suction

D

Deflate the tracheal cuff

Question 16 Explanation: 

Before deflating the tracheal cuff (D), the nurse will apply oral or nasal suction to the airway to prevent secretions from falling into the lung. Dressing change (A) and humidity (B) do not relate to suctioning.

Question 17

A 58-year-old client with a 40-year history of smoking one to two packs of cigarettes a day has a chronic cough producing thick sputum, peripheral edema, and cyanotic nail beds. Based on this information, he most likely has which of the following conditions?

A

Adult respiratory distress syndrome (ARDS)

B

Asthma

C

Chronic obstructive bronchitis

D

Emphysema

Question 17 Explanation: 

Because of his extensive smoking history and symptoms, the client most likely has chronic obstructive bronchitis. Clients with ARDS have acute symptoms of and typically need large amounts of oxygen. Clients with asthma and emphysema tend not to have a chronic cough or peripheral edema.

Question 18

A nurse is caring for a client with emphysema. The client is receiving oxygen. The nurse assesses the oxygen flow rate to ensure that it does not exceed

A

1 L/min

B

2 L/min

C

6 L/min

D

10 L/min

Question 18 Explanation: 

One to 3 L/min of oxygen by nasal cannula may be required to raise to PaO2 to 60 to 80 mm Hg. However, oxygen is used cautiously and should not exceed 2 L/min. Because of the long-standing hypercapnia, the respiratory drive is triggered by low oxygen levels rather than increased carbon dioxide levels, as is the case in normal respiratory system.

Question 19

Which of the following respiratory disorders is most common in the first 24 to 48 hours after surgery?

A

Atelectasis

B

Bronchitis

C

Pneumonia

D

Pneumothorax

Question 19 Explanation: 

Atelectasis develops when there’s interference with the normal negative pressure that promotes lung expansion. Clients in the postoperative phase often splint their breathing because of pain and positioning, which causes hypoxia. It’s uncommon for any of the other respiratory disorders to develop.

Question 20

Theophylline (Theo-Dur) tablets are prescribed for a client with chronic airflow limitation, and the nurse instructs the client about the medication. Which statement by the client indicates a need for further teaching?

A

“I will take the medication on an empty stomach.”

B

“I will take the medication with food.”

C

“I will continue to take the medication even if I am feeling better.”

D

“Periodic blood levels will need to be obtained.”

Question 20 Explanation: 

Theo-Dur is a bronchodilator. The medication should be administered with food such as milk and crackers to prevent GI irritation.

Question 21

A nurse is caring for a client with an ileostomy understands that the client is most at risk for developing which acid-base disorder?

A

Respiratory acidosis

B

Respiratory alkalosis

C

Metabolic acidosis

D

Metabolic alkalosis

Question 21 Explanation: 

Intestinal secretions are high in bicarbonate and may be lost through enteric drainage tubes or an ileostomy or with diarrhea (remember, diarrhea is coming out of thebase). These conditions result in metabolic acidosis.

Question 22

Pseudoephedrine (Sudafed) has been ordered as a nasal decongestant. Which of the following is a possible side effect of this drug?

A

Constipation

B

Bradycardia

C

Diplopia

D

Restlessness

Question 22 Explanation: 

Side effects of pseudoephedrine are experienced primarily in the cardiovascular system and through sympathetic effects on the CNS. The most common CNS effects include restlessness, dizziness, tension, anxiety, insomnia, and weakness. Common cardiovascular side effects include tachycardia, hypertension, palpitations, and arrhythmias. Constipation and diplopia are not side effects of pesudoephedrine. Tachycardia, not bradycardia, is a side effect of pseudoephedrine.

Question 23

Aminophylline (theophylline) is prescribed for a client with acute bronchitis. A nurse administers the medication, knowing that the primary action of this medication is to:

A

Promote expectoration

B

Suppress the cough

C

Relax smooth muscles of the bronchial airway

D

Prevent infection

Question 23 Explanation: 

Aminophylline is a bronchodilator that directly relaxes the smooth muscles of the bronchial airway.

Question 24

Dani was given dilaudid for pain. She’s sleeping and her respiratory rate is 4 breaths/minute. If action isn’t taken quickly, she might have which of the following reactions?

A

Asthma attack

B

Respiratory arrest

C

Be pissed about receiving Narcan

D

Wake up on her own

Question 24 Explanation: 

Narcotics can cause respiratory arrest if given in large quantities. Its unlikely Dani will have an asthma attack or wake up on her own. She may be pissed for a minute, but then she’d be grateful for saving her butt.

Question 25

The physician has scheduled a client for a left pneumonectomy. The position that will most likely be ordered postoperatively for his is the:

A

Unoperative side or back

B

Operative side or back

C

Back only

D

Back or either side.

Question 25 Explanation: 

Positioning the client on the operative side facilitates the accumulation of serisanguineous fluid. The fluid forms a solid mass, which prevents the remaining lung from being drawn into the space.

Question 26

A client has started a new drug for hypertension. Thirty minutes after he takes the drug, he develops chest tightness and becomes short of breath and tachypneic. He has a decreased level of consciousness. These signs indicate which of the following conditions?

A

Asthma attack

B

Pulmonary embolism

C

respiratory failure

D

Rheumatoid arthritis

Question 26 Explanation: 

The client was reacting to the drug with respiratory signs of impending anaphylaxis, which could lead to eventual respiratory failure. Although the signs are also related to an asthma attack or a pulmonary embolism, consider the new drug first. Rheumatoid arthritis doesn’t manifest these signs.

Question 27

The term “pink puffer” refers to the client with which of the following conditions?

A

ARDS

B

Asthma

C

Chronic obstructive bronchitis

D

Emphysema

Question 27 Explanation: 

Because of the large amount of energy it takes to breathe, clients with emphysema are usually cachectic. They’re pink and usually breathe through pursed lips, hence the term “puffer”. Clients with ARDS are usually acutely short of breath. Clients with asthma don’t have any particular characteristics, and clients with chronic obstructive bronchitis are bloated and cyanotic in appearance.

Question 28

A client has been taking flunisolide (Aerobid), two inhalations a day, for treatment of asthma. He tells the nurse that he has painful, white patches in his mouth. Which response by the nurse would be the most appropriate?

A

“This is an anticipated side-effect of your medication. It should go away in a couple of weeks.”

B

“You are using your inhaler too much and it has irritated your mouth.”

C

“You have developed a fungal infection from your medication. It will need to be treated with an antibiotic.”

D

“Be sure to brush your teeth and floss daily. Good oral hygiene will treat this problem.”

Question 28 Explanation: 

Use of oral inhalant corticosteroids, such as flunisolide, can lead to the development of oral thrush, a fungal infection. Once developed, thrush must be treated by antibiotic therapy; it will not resolve on its own. Fungal infections can develop even without overuse of the Corticosteroid inhaler. Although good oral hygiene can help prevent development of a fungal infection, it cannot be used alone to treat the problem.

Question 29

A 66-year-old client has marked dyspnea at rest, is thin, and uses accessory muscles to breathe. He’s tachypneic, with a prolonged expiratory phase. He has no cough. He leans forward with his arms braced on his knees to support his chest and shoulders for breathing. This client has symptoms of which of the following respiratory disorders?

A

ARDS

B

Asthma

C

Chronic obstructive bronchitis

D

Emphysema

Question 29 Explanation: 

These are classic signs and symptoms of a client with emphysema. Clients with ARDS are acutely short of breath and require emergency care; those with asthma are also acutely short of breath during an attack and appear very frightened. Clients with chronic obstructive bronchitis are bloated and cyanotic in appearance.

Question 30

Following the initial care of a client with asthma and impending anaphylaxis from hypersensitivity to a drug, the nurse should take which of the following steps next?

A

Administer beta-adrenergic blockers

B

Administer bronchodilators

C

Obtain serum electrolyte levels

D

Have the client lie flat in the bed.

Question 30 Explanation: 

Bronchodilators would help open the client’s airway and improve his oxygenation status. Beta-adrenergic blockers aren’t indicated in the management of asthma because they may cause bronchospasm. Obtaining laboratory values wouldn’t be done on an emergency basis, and having the client lie flat in bed could worsen his ability to breathe.

Question 31

Auscultation of a client’s lungs reveals crackles in the left posterior base. The nursing intervention is to:

A

Repeat auscultation after asking the client to deep breathe and cough.

B

Instruct the client to limit fluid intake to less than 2000 ml/day.

C

Inspect the client’s ankles and sacrum for the presence of edema

D

Place the client on bedrest in a semi-Fowlers position.

Question 31 Explanation: 

Although crackles often indicate fluid in the alveoli, they may also be related to hypoventilation and will clear after a deep breath or a cough. It is, therefore, premature to impose fluid (B) or activity (D) restrictions . Inspection for edema (C) would be appropriate after reauscultation.

Question 32

The nurse is planning to teach a client with COPD how to cough effectively. Which of the following instructions should be included?

A

Take a deep abdominal breath, bend forward, and cough 3 to 4 times on exhalation.

B

Lie flat on back, splint the thorax, take two deep breaths and cough.

C

Take several rapid, shallow breaths and then cough forcefully.

D

Assume a side-lying position, extend the arm over the head, and alternate deep breathing with coughing.

Question 32 Explanation: 

The goal of effective coughing is to conserve energy, facilitate removal of secretions, and minimize airway collapse. The client should assume a sitting position with feet on the floor if possible. The client should bend forward slightly and, using pursed-lip breathing, exhale. After resuming an upright position, the client should use abdominal breathing to slowly and deeply inhale. After repeating this process 3 or 4 times, the client should take a deep abdominal breath, bend forward and cough 3 or 4 times upon exhalation (“huff” cough). Lying flat does not enhance lung expansion; sitting upright promotes full expansion of the thorax. Shallow breathing does not facilitate removal of secretions, and forceful coughing promotes collapse of airways. A side-lying position does not allow for adequate chest expansion to promote deep breathing.

Question 33

A client has been taking benzonatate (Tessalin Perles) as prescribed. A nurse concludes that the medication is having the intended effect if the client experiences:

A

Decreased anxiety level

B

Increased comfort level

C

Reduction of N/V

D

Decreased frequency and intensity of cough

Question 33 Explanation: 

Benzonatate is a locally acting antitussive the effectiveness of which is measured by the degree to which it decreases the intensity and frequency of cough without eliminating the cough reflex.

Question 34

A client’s ABG results are as follows: pH: 7.16; PaCO2 80 mm Hg; PaO2 46 mm Hg; HCO3– 24 mEq/L; SaO2 81%. This ABG result represents which of the following conditions?

A

Metabolic acidosis

B

Metabolic alkalosis

C

Respiratory acidosis

D

Respiratory alkalosis

Question 34 Explanation: 

You all should know this. Practice some problems if you got this wrong.

Question 35

Which of the following health promotion activities should the nurse include in the discharge teaching plan for a client with asthma?

A

Incorporate physical exercise as tolerated into the treatment plan.

B

Monitor peak flow numbers after meals and at bedtime.

C

Eliminate stressors in the work and home environment

D

Use sedatives to ensure uninterrupted sleep at night.

Question 35 Explanation: 

Physical exercise is beneficial and should be incorporated as tolerated into the client’s schedule. Peak flow numbers should be monitored daily, usually in the morning (before taking medication). Peak flow does not need to be monitored after each meal. Stressors in the client’s life should be modified but cannot be totally eliminated. Although adequate sleep is important, it is not recommended that sedatives be routinely taken to induce sleep.

Question 36

A client is in danger of respiratory arrest following the administration of a narcotic analgesic. An arterial blood gas value is obtained. The nurse would expect to PaCO2 to be which of the following values?

A

15 mm Hg

B

30 mm Hg

C

40 mm Hg

D

80 mm Hg

Question 36 Explanation: 

A client about to go into respiratory arrest will have inefficient ventilation and will be retaining carbon dioxide. The value expected would be around 80 mm Hg. All other values are lower than expected.

Question 37

A nurse is suctioning fluids from a client via a tracheostomy tube. When suctioning, the nurse must limit the suctioning to a maximum of:

A

5 seconds

B

10 seconds

C

30 seconds

D

1 minute

Question 37 Explanation: 

Hypoxemia can be caused by prolonged suctioning, which stimulates the pacemaker cells within the heart. A vasovagal response may occurm causing bradycardia. The nurse must preoxygenate the client before suctioning and limit the suctioning pass to 10 seconds.

Question 38

A nurse teaches a client about the use of a respiratory inhaler. Which action by the client indicated a need for further teaching?

A

Removes the cap and shakes the inhaler well before use.

B

Presses the canister down with finger as he breathes in.

C

Inhales the mist and quickly exhales.

D

Waits 1 to 2 minutes between puffs if more than one puff has been prescribed.

Question 38 Explanation: 

The client should be instructed to hold his or her breath at least 10 to 15 seconds before exhaling the mist.

Question 39

An oxygenated delivery system is prescribed for a client with COPD to deliver a precise oxygen concentration. Which of the following types of oxygen delivery systems would the nurse anticipate to be prescribed?

A

Venturi mask

B

Aerosol mask

C

Face tent

D

Tracheostomy collar

Question 39 Explanation: 

The venture mask delivers the most accurate oxygen concentration. The Venturi mask is the best oxygen delivery system for the client with chronic airflow limitation because it delivers a precise oxygen concentration. The face tent, the aerosol mask, and the tracheostomy collar are also high-flow oxygen delivery systems but most often are used to administer high humidity.

Question 40

A female client is scheduled to have a chest radiograph. Which of the following questions is of most importance to the nurse assessing this client?

A

“Is there any possibility that you could be pregnant?”

B

“Are you wearing any metal chains or jewelry?”

C

“Can you hold your breath easily?”

D

“Are you able to hold your arms above your head?”

Question 40 Explanation: 

The most important item to ask about is the client’s pregnancy status because pregnant women should not be exposed to radiation. Clients are also asked to remove any chains or metal objects that could interfere with obtaining an adequate film. A chest radiograph most often is done at full inspiration, which gives optimal lung expansion. If a lateral view of the chest is ordered, the client is asked to raise the arms above the head. Most films are done in posterior-anterior view.

Question 41

A 34-year-old woman with a history of asthma is admitted to the emergency department. The nurse notes that the client is dyspneic, with a respiratory rate of 35 breaths/minute, nasal flaring, and use of accessory muscles. Auscultation of the lung fields reveals greatly diminished breath sounds. Based on these findings, what action should the nurse take to initiate care of the client?

A

Initiate oxygen therapy and reassess the client in 10 minutes.

B

Draw blood for an ABG analysis and send the client for a chest x-ray.

C

Encourage the client to relax and breathe slowly through the mouth

D

Administer bronchodilators

Question 41 Explanation: 

In an acute asthma attack, diminished or absent breath sounds can be an ominous sign of indicating lack of air movement in the lungs and impending respiratory failure. The client requires immediate intervention with inhaled bronchodilators, intravenous corticosteroids, and possibly intravenous theophylline. Administering oxygen and reassessing the client 10 minutes later would delay needed medical intervention, as would drawing an ABG and obtaining a chest x-ray. It would be futile to encourage the client to relax and breathe slowly without providing necessary pharmacologic intervention.

Question 42

Which of the following assessment findings would help confirm a diagnosis of asthma in a client suspected of having the disorder?

A

Circumoral cyanosis

B

Increased forced expiratory volume

C

Inspiratory and expiratory wheezing

D

Normal breath sounds

Question 42 Explanation: 

Inspiratory and expiratory wheezes are typical findings in asthma. Circumoral cyanosis may be present in extreme cases of respiratory distress. The nurse would expect the client to have a decreased forced expiratory volume because asthma is an obstructive pulmonary disease. Breath sounds will be “tight” sounding or markedly decreased; they won’t be normal.

Question 43

The term “blue bloater” refers to which of the following conditions?

A

Adult respiratory distress syndrome (ARDS)

B

Asthma

C

Chronic obstructive bronchitis

D

Emphysema

Question 43 Explanation: 

Clients with chronic obstructive bronchitis appear bloated; they have large barrel chests and peripheral edema, cyanotic nail beds and, at times, circumoral cyanosis. Clients with ARDS are acutely short of breath and frequently need intubation for mechanical ventilation and large amounts of oxygen. Clients with asthma don’t exhibit characteristics of chronic disease, and clients with emphysema appear pink and cachectic (a state of ill health, malnutrition, and wasting).

Question 44

A nurse is suctioning fluids from a client through an endotracheal tube. During the suctioning procedure, the nurse notes on the monitor that the heart rate decreases. Which of the following is the most appropriate nursing intervention?

A

Continue to suction

B

Ensure that the suction is limited to 15 seconds

C

Stop the procedure and reoxyenate the client

D

Notify the physician immediately.

Question 44 Explanation: 

During suctioning, the nurse should monitor the client closely for side effects, including hypoxemia, cardiac irregularities such as a decrease in HR resulting from vagal stimulation, mucousal trauma, hypotension, and paroxysmal coughing. If side effects develop, especially cardiac irregularities, this procedure is stopped and the client is reoxygenated.

Question 45

Emergency treatment for a client with impending anaphylaxis secondary to hypersensitivity to a drug should include which of the following actions first?

A

Administering oxygen

B

Inserting an I.V. catheter

C

Obtaining a complete blood count (CBC)

D

Taking vital signs

Question 45 Explanation: 

Giving oxygen would be the best first action in this case. Vital signs then should be checked and the physician immediately notified. If the client doesn’t already have an I.V. catheter, one may be inserted now if anaphylactic shock is developing. Obtaining a CBC wouldn’t help the emergency situation.

Question 46

Which of the following types of asthma involves an acute asthma attack brought on by an upper respiratory infection?

A

Emotional

B

Extrinsic

C

Intrinsic

D

Mediated

Question 46 Explanation: 

Intrinsic asthma doesn’t have an easily identifiable allergen and can be triggered by the common cold. Asthma caused be emotional reasons is considered to be in the extrinsic category. Extrinsic asthma is caused by dust, molds, and pets; easily identifiable allergens. Mediated asthma doesn’t exist.

Question 47

A nurse is preparing to obtain a sputum specimen from a client. Which of the following nursing actions will facilitate obtaining the specimen?

A

Limiting fluids

B

Having the client take 3 deep breaths.

C

Asking the client to spit into the collection container.

D

Asking the client to obtain the specimen after eating.

Question 47 Explanation: 

To obtain a sputum specimen, the client should rinse the mouth to prevent contamination, breathe deeply, and then cough unto a sputum specimen container. The client should be encouraged to cough and not spit so as to obtain sputum. Sputum can be thinned by fluids or by a respiratory treatment such as inhalation of nebulized saline or water. The optimal time to obtain a specimen is on arising in the morning.

Question 48

The nurse is teaching the client how to use a metered dose inhaler (MDI) to administer a Corticosteroid drug. Which of the following client actions indicates that he us using the MDI correctly? Select all that apply.

A

The inhaler is held upright.

B

Head is tilted down while inhaling the medication

C

Client waits 5 minutes between puffs.

D

Mouth is rinsed with water following administration

E

Client lies supine for 15 minutes following administration.

Question 49

The most reliable index to determine the respiratory status of a client is to:

A

Observe the chest rising and falling

B

Observe the skin and mucous membrane color.

C

Listen and feel the air movement.

D

Determine the presence of a femoral pulse.

Question 49 Explanation: 

To check for breathing, the nurse places her ear and cheek next to the client’s mouth and nose to listen and feel for air movement. The chest rising and falling (A) is not conclusive of a patent airway. Observing skin color (B) is not an accurate assessment of respiratory status, nor is checking the femoral pulse.

Question 50

An acceleration in oxygen dissociation from hemoglobin, and thus oxygen delivery to the tissues, is caused by:

A

A decreasing oxygen pressure in the blood

B

An increasing carbon dioxide pressure in the blood

C

A decreasing oxygen pressure and/or an increasing carbon dioxide pressure in the blood.

D

An increasing oxygen pressure and/or a decreasing carbon dioxide pressure in the blood.

Question 50 Explanation: 

The lower the PO2 and the higher the PCO2, the more rapidly oxygen dissociated from the oxy-hemoglobin molecule.

Question 51

The nurse reviews the ABG values of a client. The results indicate respiratory acidosis. Which of the following values would indicate that this acid-base imbalance exists?

A

pH of 7.48

B

PCO2 of 32 mm Hg

C

pH of 7.30

D

HCO3– of 20 mEq/L

Question 52

A client is scheduled for blood to be drawn from the radial artery for an ABG determination. Before the blood is drawn, an Allen’s test is performed to determine the adequacy of the:

A

Popliteal circulation

B

Ulnar circulation

C

Femoral circulation

D

Carotid circulation

Question 52 Explanation: 

Before radial puncture for obtaining an ABG, you should perform an Allen’s test to determine adequate ulnar circulation. Failure to determine the presence of adequate collateral circulation could result in severe ischemic injury o the hand if damage to the radial artery occurs with arterial puncture.

Question 53

Which of the following measures can reduce or prevent the incidence of atelectasis in a post-operative client?

A

Chest physiotherapy

B

Mechanical ventilation

C

Reducing oxygen requirements

D

Use of an incentive spirometer

Question 53 Explanation: 

Using an incentive spirometer requires the client to take deep breaths and promotes lung expansion. Chest physiotherapy helps mobilize secretions but won’t prevent atelectasis. Reducing oxygen requirements or placing someone on mechanical ventilation doesn’t affect the development of atelectasis.

Question 54

A nurse reviews the ABG values and notes a pH of 7.50, a PCO2 of 30 mm Hg, and an HCO3 of 25 mEq/L. The nurse interprets these values as indicating:

A

Respiratory acidosis uncompensated

B

Respiratory alkalosis uncompensated

C

Metabolic acidosis uncompensated

D

Metabolic acidosis partially compensated.

Question 54 Explanation: 

In respiratory alkalosis the pH will be higher than normal and the PCO2 will be low.

Question 55

Which of the following treatment goals is best for the client with status asthmaticus?

A

Avoiding intubation

B

Determining the cause of the attack

C

Improving exercise tolerance

D

Reducing secretions

Question 55 Explanation: 

Inhaled beta-adrenergic agents, I.V. corticosteroids, and supplemental oxygen are used to reduce bronchospasm, improve oxygenation, and avoid intubation. Determining the trigger for the client’s attack and improving exercise tolerance are later goals. Typically, secretions aren’t a problem in status asthmaticus.

Question 56

A nurse is caring for a client with diabetic ketoacidosis and documents that the client is experiencing Kussmaul’s respirations. Based on this documentation, which of the following did the nurse observe?

A

Respirations that are abnormally deep, regular, and increased in rate.

B

Respirations that are regular but abnormally slow.

C

Respirations that are labored and increased in depth and rate

D

Respirations that cease for several seconds.

Question 56 Explanation: 

Kussmaul’s respirations are abnormally deep, regular, and increased in rate.

Question 57

Exercise has which of the following effects on clients with asthma, chronic bronchitis, and emphysema?

A

It enhances cardiovascular fitness.

B

It improves respiratory muscle strength.

C

It reduces the number of acute attacks.

D

It worsens respiratory function and is discouraged.

Question 57 Explanation: 

Exercise can improve cardiovascular fitness and help the client tolerate periods of hypoxia better, perhaps reducing the risk of heart attack. Most exercise has little effect on respiratory muscle strength, and these clients can’t tolerate the type of exercise necessary to do this. Exercise won’t reduce the number of acute attacks. In some instances, exercise may be contraindicated, and the client should check with his physician before starting any exercise program.

Question 58

A 7-year-old client is brought to the E.R. He’s tachypneic and afebrile and has a respiratory rate of 36 breaths/minute and a nonproductive cough. He recently had a cold. From his history, the client may have which of the following?

A

Acute asthma

B

Bronchial pneumonia

C

Chronic obstructive pulmonary disease (COPD)

D

Emphysema

Question 58 Explanation: 

Based on the client’s history and symptoms, acute asthma is the most likely diagnosis. He’s unlikely to have bronchial pneumonia without a productive cough and fever and he’s too young to have developed COPD or emphysema.

Question 59

A client with allergic rhinitis asks the nurse what he should do to decrease his symptoms. Which of the following instructions would be appropriate for the nurse to give the client?

A

“Use your nasal decongestant spray regularly to help clear your nasal passages.”

B

“Ask the doctor for antibiotics. Antibiotics will help decrease the secretion.”

C

“It is important to increase your activity. A daily brisk walk will help promote drainage.”

D

“Keep a diary if when your symptoms occur. This can help you identify what precipitates your attacks.”

Question 59 Explanation: 

It is important for clients with allergic rhinitis to determine the precipitating factors so that they can be avoided. Keeping a diary can help identify these triggers. Nasal decongestant sprays should not be used regularly because they can cause a rebound effect. Antibiotics are not appropriate. Increasing activity will not control the client’s symptoms; in fact, walking outdoors may increase them if the client is allergic to pollen.

Question 60

A nurse reviews the arterial blood gas results of a patient and notes the following: pH 7.45; PCO2 30 mm Hg; and bicarbonate concentration of 22 mEq/L. The nurse analyzes these results as indicating:

A

Metabolic acidosis, compensated.

B

Metabolic alkalosis, uncompensated.

C

Respiratory alkalosis, compensated.

D

Respiratory acidosis, compensated.

Question 60 Explanation: 

The normal pH is 7.35 to 7.45. In a respiratory condition, an opposite (see-saw) will be seen between the pH and the PCO2. In this situation, the pH is at the high end of the normal value and the PCO2 is low. In an alkalotic condition, the pH is up. Therefore, the values identified in the question indicate a respiratory alkalosis. Compensation occurs when the pH returns to a normal value. Because the pH is in the normal range at the high end, compensation has occurred.

Question 61

A client with COPD has developed secondary polycythemia. Which nursing diagnosis would be included in the plan of care because of the polycythemia?

A

Fluid volume deficit related to blood loss.

B

Impaired tissue perfusion related to thrombosis

C

Activity intolerance related to dyspnea

D

Risk for infection related to suppressed immune response.

Question 61 Explanation: 

Chronic hypoxia associated with COPD may stimulate excessive RBC production (polycythemia). This results in increased blood viscosity and the risk of thrombosis. The other nursing diagnoses are not applicable in this situation.

Question 62

The nurse teaches a client with COPD to assess for s/s of right-sided heart failure. Which of the following s/s would be included in the teaching plan?

A

Clubbing of nail beds

B

Hypertension

C

Peripheral edema

D

Increased appetite

Question 62 Explanation: 

Right-sided heart failure is a complication of COPD that occurs because of pulmonary hypertension. Signs and symptoms of right-sided heart failure include peripheral edema, jugular venous distention, hepatomegaly, and weight gain due to increased fluid volume. Clubbing of nail beds is associated with conditions of chronic hypoxia. Hypertension is associated with left-sided heart failure. Clients with heart failure have decreased appetites.

Question 63

Basilar crackles are present in a client’s lungs on auscultation. The nurse knows that these are discrete, noncontinuous sounds that are:

A

Caused by the sudden opening of alveoli

B

Usually more prominent during expiration

C

Produced by airflow across passages narrowed by secretions

D

Found primarily in the pleura.

Question 63 Explanation: 

Basilar crackles are usually heard during inspiration and are caused by sudden opening of the alveoli.

Question 64

Which of the following is a priority goal for the client with COPD?

A

Maintaining functional ability

B

Minimizing chest pain

C

Increasing carbon dioxide levels in the blood

D

Treating infectious agents

Question 64 Explanation: 

A priority goal for the client with COPD is to manage the s/s of the disease process so as to maintain the client’s functional ability. Chest pain is not a typical sign of COPD. The carbon dioxide concentration in the blood is increased to an abnormal level in clients with COPD; it would not be a goal to increase the level further. Preventing infection would be a goal of care for the client with COPD.

Question 65

A client states that the physician said the tidal volume is slightly diminished and asks the nurse what this means. The nurse explains that the tidal volume is the amount of air:

A

Exhaled forcibly after a normal expiration

B

Exhaled after there is a normal inspiration

C

Trapped in the alveoli that cannot be exhaled

D

Forcibly inspired over and above a normal respiration.

Question 65 Explanation: 

Tidal volume (TV) is defined as the amount of air exhaled after a normal inspiration.

Question 66

A 19-year-old comes into the emergency department with acute asthma. His respiratory rate is 44 breaths/minute, and he appears to be in acute respiratory distress. Which of the following actions should be taken first?

A

Take a full medication history

B

Give a bronchodilator by neubulizer

C

Apply a cardiac monitor to the client

D

Provide emotional support to the client.

Question 66 Explanation: 

The client is having an acute asthma attack and needs to increase oxygen delivery to the lung and body. Nebulized bronchodilators open airways and increase the amount of oxygen delivered. First resolve the acute phase of the attack ad how to prevent attacks in the future. It may not be necessary to place the client on a cardiac monitor because he’s only 19-years-old, unless he has a past medical history of cardiac problems.

Question 67

When developing a discharge plan to manage the care of a client with COPD, the nurse should anticipate that the client will do which of the following?

A

Develop infections easily

B

Maintain current status

C

Require less supplemental oxygen

D

Show permanent improvement.

Question 67 Explanation: 

A client with COPD is at high risk for development of respiratory infections. COPD is a slowly progressive; therefore, maintaining current status and establishing a goal that the client will require less supplemental oxygen are unrealistic expectations. Treatment may slow progression of the disease, but permanent improvement is highly unlikely.

Question 68

If a client continues to hypoventilate, the nurse will continually assess for a complication of:

A

Respiratory acidosis

B

Respiratory alkalosis

C

Metabolic acidosis

D

Metabolic alkalosis

Question 68 Explanation: 

Respiratory acidosis represents an increase in the acid component, carbon dioxide, and an increase in the hydrogen ion concentration (decreased pH) of the arterial blood.

Question 69

Clients with chronic obstructive bronchitis are given diuretic therapy. Which of the following reasons best explains why?

A

Reducing fluid volume reduces oxygen demand.

B

Reducing fluid volume improves clients’ mobility.

C

Restricting fluid volume reduces sputum production.

D

Reducing fluid volume improves respiratory function.

Question 69 Explanation: 

Reducing fluid volume reduces the workload of the heart, which reduces oxygen demand and, in turn, reduces the respiratory rate. It may also reduce edema and improve mobility a little, but exercise tolerance will still be harder to clear airways. Reducing fluid volume won’t improve respiratory function, but may improve oxygenation.

Question 70

A client’s arterial blood gas levels are as follows: pH 7.31; PaO2 80 mm Hg, PaCO2 65 mm Hg; HCO3– 36 mEq/L. Which of the following signs or symptoms would the nurse expect?

A

Cyanosis

B

Flushed skin

C

Irritability

D

Anxiety

Question 70 Explanation: 

The high PaCO2 level causes flushing due to vasodilation. The client also becomes drowsy and lethargic because carbon dioxide has a depressant effect on the CNS. Cyanosis is a late sign of hypoxia. Irritability and anxiety are not common with a PaCO2 level of 65 mm Hg but are associated with hypoxia.

Question 71

It’s highly recommended that clients with asthma, chronic bronchitis, and emphysema have Pneumovax and flu vaccinations for which of the following reasons?

A

All clients are recommended to have these vaccines

B

These vaccines produce bronchodilation and improve oxygenation.

C

These vaccines help reduce the tachypnea these clients experience.

D

Respiratory infections can cause severe hypoxia and possibly death in these clients.

Question 71 Explanation: 

It’s highly recommended that clients with respiratory disorders be given vaccines to protect against respiratory infection. Infections can cause these clients to need intubation and mechanical ventilation, and it may be difficult to wean these clients from the ventilator. The vaccines have no effect on bronchodilation or respiratory care.

Question 72

A client with COPD reports steady weight loss and being “too tired from just breathing to eat.” Which of the following nursing diagnoses would be most appropriate when planning nutritional interventions for this client?

A

Altered nutrition: Less than body requirements related to fatigue.

B

Activity intolerance related to dyspnea.

C

Weight loss related to COPD.

D

Ineffective breathing pattern related to alveolar hypoventilation.

Question 72 Explanation: 

The client’s problem is altered nutrition—specifically, less than required. The cause, as stated by the client, is the fatigue associated with the disease process. Activity intolerance is a likely diagnosis but is not related to the client’s nutritional problems. Weight loss is not a nursing diagnosis. Ineffective breathing pattern may be a problem, but this diagnosis does not specifically address the problem of weight loss described by the client.

Question 73

Lisa is newly diagnosed with asthma and is being discharged from the hospital after an episode of status asthmaticus. Discharge teaching should include which of the following:

A

Limitations in sports that will be imposed by the illness

B

Specific instructions on staying cal during an attack

C

The relationship of symptoms and a specific trigger such as physical exercise

D

Incidence of status asthmaticus in children and teens

Question 73 Explanation: 

COPD clients have low oxygen and high carbon dioxide levels. Therefore, hypoxia is the main stimulus for ventilation is persons with chronic hypercapnea. Increasing the level of oxygen would decrease the stimulus to breathe.

Question 74

When teaching a client with COPD to conserve energy, the nurse should teach the client to lift objects:

A

While inhaling through an open mouth.

B

While exhaling through pursed lips

C

After exhaling but before inhaling.

D

While taking a deep breath and holding it.

Question 74 Explanation: 

Exhaling requires less energy than inhaling. Therefore, lifting while exhaling saves energy and reduced perceived dyspnea. Pursing the lips prolongs exhalation and provides the client with more control over breathing. Lifting after exhalation but before inhaling is similar to lifting with the breath held. This should not be recommended because it is similar to the Valsalva maneuver, which can stimulate cardiac dysrhythmias.

Question 75

A client is prescribed metaproterenol (Alupent) via a metered dose inhaler (MDI), two puffs every 4 hours. The nurse instructs the client to report side effects. Which of the following are potential side effects of metaproterenol?

A

Irregular heartbeat

B

Constipation

C

Petal edema

D

Decreased heart rate.

Question 75 Explanation: 

Irregular heart rates should be reported promptly to the care provider. Metaproterenol may cause irregular heartbeat, tachycardia, or anginal pain because of its adrenergic effect on the beta-adrenergic receptors in the heart. It is not recommended for use in clients with known cardiac disorders. Metaproterenol does not cause constipation, petal edema, or bradycardia.

Question 76

Which of the following is the primary reason to teach pursed-lip breathing to clients with emphysema?

A

To promote oxygen intake

B

To strengthen the diaphragm

C

To strengthen the intercostal muscles

D

To promote carbon dioxide elimination

Question 76 Explanation: 

Pursed lip breathing prolongs exhalation and prevents air trapping in the alveoli, thereby promoting carbon dioxide elimination. By prolonged exhalation and helping the client relax, pursed-lip breathing helps the client learn to control the rate and depth of respiration. Pursed-lip breathing does not promote the intake of oxygen, strengthen the diaphragm, or strengthen intercostal muscles.

Question 77

A nurse plans care for a client with chronic obstructive pulmonary disease, knowing that the client is most likely to experience what type of acid-base imbalance?

A

Respiratory acidosis

B

Respiratory alkalosis

C

Metabolic acidosis

D

Metabolic alkalosis

Question 77 Explanation: 

Respiratory acidosis is most often due to hypoventilation. Chronic respiratory acidosis is most commonly caused by COPD. In end-stage disease, pathological changes lead to airway collapse, air trapping, and disturbance of ventilation-perfusion relationships.

Question 78

A nurse is caring for a client who is on a mechanical ventilator. Blood gas results indicate a pH of 7.50 and a PCO2 of 30 mm Hg. The nurse has determined that the client is experiencing respiratory alkalosis. Which laboratory value would most likely be noted in this condition?

A

Sodium level of 145 mEq/L

B

Potassium level of 3.0 mEq/L

C

Magnesium level of 2.0 mg/L

D

Phosphorus level of 4.0 mg/dl

Question 78 Explanation: 

Clinical manifestations of respiratory alkalosis include headache, tachypnea, paresthesias, tetany, vertigo, convulsions, hypokalemia, and hypocalcemia. Options A, C, and D identify normal laboratory values. Option B identifies the presence of hypokalemia.

Question 79

The nurse assesses the respiratory status of a client who is experiencing an exacerbation of COPD secondary to an upper respiratory tract infection. Which of the following findings would be expected?

A

Normal breath sounds

B

Prolonged inspiration

C

Normal chest movement

D

Coarse crackles and rhonchi

Question 79 Explanation: 

Exacerbations of COPD are frequently caused by respiratory infections. Coarse crackles and rhonchi would be auscultated as air moves through airways obstructed with secretions. In COPD, breath sounds are diminished because of an enlarged anteroposterior diameter of the chest. Expiration, not inspiration, becomes prolonged. Chest movement is decreased as lungs become overdistended.

Question 80

A 69-year-old client appears thin and cachectic. He’s short of breath at rest and his dyspnea increases with the slightest exertion. His breath sounds are diminished even with deep inspiration. These signs and symptoms fit which of the following conditions?

A

ARDS

B

Asthma

C

Chronic obstructive bronchitis

D

Emphysema

Question 80 Explanation: 

In emphysema, the wall integrity of the individual air sacs is damaged, reducing the surface area available for gas exchange. Very little air movement occurs in the lungs because of bronchiole collapse, as well. In ARDS, the client’s condition is more acute and typically requires mechanical ventilation. In asthma and bronchitis, wheezing is prevalent.

Question 81

Which of the following pathophysiological mechanisms that occurs in the lung parenchyma allows pneumonia to develop?

A

Atelectasis

B

Bronchiectasis

C

Effusion

D

Inflammation

Question 81 Explanation: 

The most common feature of all types of pneumonia is an inflammatory pulmonary response to the offending organism or agent. Atelectasis and brochiectasis indicate a collapse of a portion of the airway that doesn’t occur with pneumonia. An effusion is an accumulation of excess pleural fluid in the pleural space, which may be a secondary response to pneumonia.

Question 82

The client with asthma should be taught that which of the following is one of the most common precipitating factors of an acute asthma attack?

A

Occupational exposure to toxins

B

Viral respiratory infections

C

Exposure to cigarette smoke

D

Exercising in cold temperatures

Question 82 Explanation: 

The most common precipitator of asthma attacks is viral respiratory infection. Clients with asthma should avoid people who have the flu or a cold and should get yearly flu vaccinations. Environmental exposure to toxins or heavy particulate matter can trigger asthma attacks; however, far fewer asthmatics are exposed to such toxins than are exposed to viruses. Cigarette smoke can also trigger asthma attacks, but to a lesser extent than viral respiratory infections. Some asthmatic attacks are triggered by exercising in cold weather.

Question 83

An elderly client with pneumonia may appear with which of the following symptoms first?

A

Altered mental status and dehydration

B

fever and chills

C

Hemoptysis and dyspnea

D

Pleuretic chest pain and cough

Question 83 Explanation: 

Fever, chills, hemoptysis, dyspnea, cough, and pleuric chest pain are the common symptoms of pneumonia, but elderly clients may first appear with only an altered mental status and dehydration due to a blunted immune response.

Question 84

Assessing a client who has developed atelectasis postoperatively, the nurse will most likely find:

A

A flushed face

B

Dyspnea and pain

C

Decreased temperature

D

Severe cough and no pain.

Question 84 Explanation: 

Atelectasis is a collapse of the alveoli due to obstruction or hypoventilation. Clients become short of breath, have a high temperature, and usually experience severe pain but do not have a severe cough (D). The shortness of breath is a result of decreased oxygen-carbon dioxide exchange at the alveolar level.

Question 85

A nurse instructs a client to use the pursed lip method of breathing. The client asks the nurse about the purpose of this type of breathing. The nurse responds, knowing that the primary purpose of pursed lip breathing is:

A

Promote oxygen intake

B

Strengthen the diaphragm

C

Strengthen the intercostal muscles

D

Promote carbon dioxide elimination

Question 85 Explanation: 

Pursed lip breathing facilitates maximum expiration for clients with obstructive lung disease. This type of breathing allows better expiration by increasing airway pressure that keeps air passages open during exhalation.

Question 86

Emergency treatment of a client in status asthmaticus includes which of the following medications?

A

Inhaled beta-adrenergic agents

B

Inhaled corticosteroids

C

I.V. beta-adrenergic agents

D

Oral corticosteroids

Question 86 Explanation: 

Inhaled beta-adrenergic agents help promote bronchodilation, which improves oxygenation. I.V. beta-adrenergic agents can be used but have to be monitored because of their greater systemic effects. They’re typically used when the inhaled beta-adrenergic agents don’t work. Corticosteriods are slow-acting, so their use won’t reduce hypoxia in the acute phase.

Question 87

Which of the following additional assessment data should immediately be gathered to determine the status of a client with a respiratory rate of 4 breaths/minute?

A

Arterial blood gas (ABG) and breath sounds

B

Level of consciousness and a pulse oximetry value.

C

Breath sounds and reflexes

D

Pulse oximetry value and heart sounds

Question 87 Explanation: 

First, the nurse should attempt to rouse the client because this should increase the client’s respiratory rate. If available, a spot pulse oximetry check should be done and breath sounds should be checked. The physician should be notified immediately if of the findings. He’ll probably order ABG analysis to determine specific carbon dioxide and oxygen levels, which will indicate the effectiveness of ventilation. Reflexes and heart sounds will be part of the more extensive examination done after these initial actions are completed.

Question 88

A client is receiving isoetharine hydrochloride (Bronkosol) via a nebulizer. The nurse monitors the client for which side effect of this medication?

A

Constipation

B

Diarrhea

C

Bradycardia

D

Tachycardia

Question 88 Explanation: 

Side effects that can occur from a beta 2 agonist include tremors, nausea, nervousness, palpitations, tachycardia, peripheral vasodilation, and dryness of the mouth or throat.

Question 89

A client has just returned to a nursing unit following bronchoscopy. A nurse would implement which of the following nursing interventions for this client?

A

Encouraging additional fluids for the next 24 hours

B

Ensuring the return of the gag reflex before offering foods or fluids

C

Administering atropine intravenously

D

Administering small doses of midazolam (Versed).

Question 89 Explanation: 

After bronchoscopy, the nurse keeps the client on NPO status until the gag reflex returns because the preoperative sedation and the local anesthesia impair swallowing and the protective laryngeal reflexes for a number of hours. Additional fluids is unnecessary because no contrast dye is used that would need to be flushed from the system. Atropine and Versed would be administered before the procedure, not after.

Question 90

A client with emphysema should receive only 1 to 3 L/minute of oxygen, if needed, or he may lose his hypoxic drive. Which of the following statements is correct about hypoxic drive?

A

The client doesn’t notice he needs to breathe.

B

The client breathes only when his oxygen levels climb above a certain point.

C

The client breathes only when his oxygen levels dip below a certain point.

D

The client breathes only when his carbon dioxide level dips below a certain point.

Question 90 Explanation: 

Clients with emphysema breathe when their oxygen levels drop to a certain level; this is known as the hypoxic drive. They don’t take a breath when their levels of carbon dioxide are higher than normal, as do those with healthy respiratory physiology. If too much oxygen is given, the client has little stimulus to take another breath. In the meantime, his carbon dioxide levels continue to climb, and the client will pass out, leading to a respiratory arrest.

Question 91

A nurse is caring for a client hospitalized with acute exacerbation of COPD. Which of the following would the nurse expect to note on assessment of this client?

A

Increased oxygen saturation with exercise

B

Hypocapnia

C

A hyperinflated chest on x-ray film

D

A widened diaphragm noted on chest x-ray film

Question 91 Explanation: 

Clinical manifestations of COPD include hypoxemia, hypercapnia, dyspnea on exertion and at rest, oxygen desaturation with exercise, and the use of accessory muscles of respiration. Chest x-ray films reveal a hyperinflated chest and a flattened diaphragm is the disease is advanced.

Question 92

Which of the following individuals would the nurse consider to have the highest priority for receiving an influenza vaccination?

A

A 60-year-old man with a hiatal hernia

B

A 36-year-old woman with 3 children

C

A 50-year-old woman caring for a spouse with cancer

D

A 60-year-old woman with osteoarthritis

Question 92 Explanation: 

Individuals who are household members or home care providers for high-risk individuals are high-priority targeted groups for immunization against influenza to prevent transmission to those who have a decreased capacity to deal with the disease. The wife who is caring for a husband with cancer has the highest priority of the clients described.

Question 93

A client with acute asthma is prescribed short-term corticosteroid therapy. What is the rationale for the use of steroids in clients with asthma?

A

Corticosteroids promote bronchodilation

B

Corticosteroids act as an expectorant

C

Corticosteroids have an anti-inflammatory effect

D

Corticosteroids prevent development of respiratory infections.

Question 93 Explanation: 

Corticosteroids have an anti-inflammatory effect and act to decrease edema in the bronchial airways and decrease mucus secretion. Corticosteroids do not have a bronchodilator effect, act as expectorants, or prevent respiratory infections.

Question 94

A nurse understands that the excessive use of oral antacids containing bicarbonate can result in which acid-base disturbance?

A

Respiratory alkalosis

B

Respiratory acidosis

C

Metabolic acidosis

D

Metabolic alkalosis

Question 94 Explanation: 

Increases in base components occur as a result of oral or parenteral intake of bicarbonates, carbonates, acetates, citrates, or lactates. Excessive use of oral antacids containing bicarbonate can cause metabolic alkalosis.

Question 95

A nurse is assessing a client with chronic airflow limitation and notes that the client has a “barrel chest.” The nurse interprets that this client has which of the following forms of chronic airflow limitation?

A

Chronic obstructive bronchitis

B

Emphysema

C

Bronchial asthma

D

Bronchial asthma and bronchitis

Question 95 Explanation: 

The client with emphysema has hyperinflation of the alveoli and flattening of the diaphragm. These lead to increased anteroposterior diameter, which is referred to as “barrel chest.” The client also has dyspnea with prolonged expiration and has hyperresonant lungs to percussion.

Question 96

Guaifenesin 300 mg four times daily has been ordered as an expectorant. The dosage strength of the liquid is 200mg/5ml. How many mL should the nurse administer each dose?

A

5.0 ml

B

7.5 ml

C

9.5 ml

D

10 ml

Question 97

A nurse is caring for a client with renal failure. Blood gas results indicate a pH of 7.30; a PCO2 of 32 mm Hg, and a bicarbonate concentration of 20 mEq/L. The nurse has determined that the client is experiencing metabolic acidosis. Which of the following laboratory values would the nurse expect to note?

A

Sodium level of 145 mEq/L

B

Magnesium level of 2.0 mg/dL

C

Potassium level of 5.2 mEq/L

D

Phosphorus level of 4.0 mg/dL

Question 97 Explanation: 

Clinical manifestations of metabolic acidosis include hyperpnea with Kussmaul’s respirations; headache; N/V, and diarrhea; fruity-smelling breath resulting from improper fat metabolism; CNS depression, including mental dullness, drowsiness, stupor, and coma; twitching, and coma. Hyperkalemia will occur.

Question 98

Teaching for a client with chronic obstructive pulmonary disease (COPD) should include which of the following topics?

A

How to have his wife learn to listen to his lungs with a stethoscope from Wal-Mart.

B

How to increase his oxygen therapy.

C

How to treat respiratory infections without going to the physician.

D

How to recognize the signs of an impending respiratory infection.

Question 98 Explanation: 

Respiratory infection in clients with a respiratory disorder can be fatal. It’s important that the client understands how to recognize the signs and symptoms of an impending respiratory infection. It isn’t appropriate for the wife to listen to his lung sounds, besides, you can’t purchase stethoscopes from Wal-Mart. If the client has signs and symptoms of an infection, he should contact his physician at once.

Question 99

A nurse is caring for a client with a nasogastric tube that is attached to low suction. The nurse monitors the client, knowing that the client is at risk for which acid-base disorder?

A

Respiratory acidosis

B

Respiratory alkalosis

C

Metabolic acidosis

D

Metabolic alkalosis

Question 99 Explanation: 

Loss of gastric fluid via nasogastric suction or vomiting causes metabolic alkalosis as a result of the loss of hydrochloric acid.

Question 100

A cyanotic client with an unknown diagnosis is admitted to the E.R. In relation to oxygen, the first nursing action would be to:

A

Wait until the client’s lab work is done.

B

Not administer oxygen unless ordered by the physician.

C

Administer oxygen at 2 L flow per minute.

D

Administer oxygen at 10 L flow per minute and check the client’s nailbeds.

Question 100 Explanation: 

Administer oxygen at 2 L/minute and no more, for if the client if emphysemic and receives too high a level of oxygen, he will develop CO2 narcosis and the respiratory system will cease to function.

Once you are finished, click the button below. Any items you have not completed will be marked incorrect. Get Results

There are 100 questions to complete.

List

Return

Shaded items are complete.

123456789101112131415161718192021222324252627282930313233343536373839404142434445464748495051525354555657585960616263646566676869707172737475767778798081828384858687888990919293949596979899100End

Return

You have completed

questions

question

Your score is

Correct

Wrong

Partial-Credit

You have not finished your quiz. If you leave this page, your progress will be lost.

Correct Answer

You Selected

Not Attempted

Final Score on Quiz

Attempted Questions Correct

Attempted Questions Wrong

Questions Not Attempted

Total Questions on Quiz

Question Details

Results

Date

Score

Hint

Time allowed

minutes

seconds

Time used

Answer Choice(s) Selected

Question Text

All done

Need more practice!

Keep trying!

Not bad!

Good work!

Perfect!

Text Mode

Text Mode – Text version of the exam

1. An elderly client with pneumonia may appear with which of the following symptoms first?

  1. Altered mental status and dehydration
  2. fever and chills
  3. Hemoptysis and dyspnea
  4. Pleuretic chest pain and cough

2.  Which of the following pathophysiological mechanisms that occurs in the lung parenchyma allows pneumonia to develop?

  1. Atelectasis
  2. Bronchiectasis
  3. Effusion
  4. Inflammation

3.  A 7-year-old client is brought to the E.R. He’s tachypneic and afebrile and has a respiratory rate of 36 breaths/minute and a nonproductive cough. He recently had a cold. From his history, the client may have which of the following?

  1. Acute asthma
  2. Bronchial pneumonia
  3. Chronic obstructive pulmonary disease (COPD)
  4. Emphysema

4.  Which of the following assessment findings would help confirm a diagnosis of asthma in a client suspected of having the disorder?

  1. Circumoral cyanosis
  2. Increased forced expiratory volume
  3. Inspiratory and expiratory wheezing
  4. Normal breath sounds

5.  Which of the following types of asthma involves an acute asthma attack brought on by an upper respiratory infection?

  1. Emotional
  2. Extrinsic
  3. Intrinsic
  4. Mediated

6.  A client with acute asthma showing inspiratory and expiratory wheezes and a decreased expiratory volume should be treated with which of the following classes of medication right away?

  1. Beta-adrenergic blockers
  2. Bronchodilators
  3. Inhaled steroids
  4. Oral steroids

7.  A 19-year-old comes into the emergency department with acute asthma. His respiratory rate is 44 breaths/minute, and he appears to be in acute respiratory distress. Which of the following actions should be taken first?

  1. Take a full medication history
  2. Give a bronchodilator by neubulizer
  3. Apply a cardiac monitor to the client
  4. Provide emotional support to the client.

8.  A 58-year-old client with a 40-year history of smoking one to two packs of cigarettes a day has a chronic cough producing thick sputum, peripheral edema, and cyanotic nail beds. Based on this information, he most likely has which of the following conditions?

  1. Adult respiratory distress syndrome (ARDS)
  2. Asthma
  3. Chronic obstructive bronchitis
  4. Emphysema

9.  The term “blue bloater” refers to which of the following conditions?

  1. Adult respiratory distress syndrome (ARDS)
  2. Asthma
  3. Chronic obstructive bronchitis
  4. Emphysema

10.  The term “pink puffer” refers to the client with which of the following conditions?

  1. ARDS
  2. Asthma
  3. Chronic obstructive bronchitis
  4. Emphysema

11.  A 66-year-old client has marked dyspnea at rest, is thin, and uses accessory muscles to breathe. He’s tachypneic, with a prolonged expiratory phase. He has no cough. He leans forward with his arms braced on his knees to support his chest and shoulders for breathing. This client has symptoms of which of the following respiratory disorders?

  1. ARDS
  2. Asthma
  3. Chronic obstructive bronchitis
  4. Emphysema

12.  It’s highly recommended that clients with asthma, chronic bronchitis, and emphysema have Pneumovax and flu vaccinations for which of the following reasons?

  1. All clients are recommended to have these vaccines
  2. These vaccines produce bronchodilation and improve oxygenation.
  3. These vaccines help reduce the tachypnea these clients experience.
  4. Respiratory infections can cause severe hypoxia and possibly death in these clients.

13.  Exercise has which of the following effects on clients with asthma, chronic bronchitis, and emphysema?

  1. It enhances cardiovascular fitness.
  2. It improves respiratory muscle strength.
  3. It reduces the number of acute attacks.
  4. It worsens respiratory function and is discouraged.

14.  Clients with chronic obstructive bronchitis are given diuretic therapy. Which of the following reasons best explains why?

  1. Reducing fluid volume reduces oxygen demand.
  2. Reducing fluid volume improves clients’ mobility.
  3. Restricting fluid volume reduces sputum production.
  4. Reducing fluid volume improves respiratory function.

15.  A 69-year-old client appears thin and cachectic. He’s short of breath at rest and his dyspnea increases with the slightest exertion. His breath sounds are diminished even with deep inspiration. These signs and symptoms fit which of the following conditions?

  1. ARDS
  2. Asthma
  3. Chronic obstructive bronchitis
  4. Emphysema

16.  A client with emphysema should receive only 1 to 3 L/minute of oxygen, if needed, or he may lose his hypoxic drive. Which of the following statements is correct about hypoxic drive?

  1. The client doesn’t notice he needs to breathe.
  2. The client breathes only when his oxygen levels climb above a certain point.
  3. The client breathes only when his oxygen levels dip below a certain point.
  4. The client breathes only when his carbon dioxide level dips below a certain point.

17.  Teaching for a client with chronic obstructive pulmonary disease (COPD) should include which of the following topics?

  1. How to have his wife learn to listen to his lungs with a stethoscope from Wal-Mart.
  2. How to increase his oxygen therapy.
  3. How to treat respiratory infections without going to the physician.
  4. How to recognize the signs of an impending respiratory infection.

18.  Which of the following respiratory disorders is most common in the first 24 to 48 hours after surgery?

  1. Atelectasis
  2. Bronchitis
  3. Pneumonia
  4. Pneumothorax

19.  Which of the following measures can reduce or prevent the incidence of atelectasis in a post-operative client?

  1. Chest physiotherapy
  2. Mechanical ventilation
  3. Reducing oxygen requirements
  4. Use of an incentive spirometer

20.  Emergency treatment of a client in status asthmaticus includes which of the following medications?

  1. Inhaled beta-adrenergic agents
  2. Inhaled corticosteroids
  3. I.V. beta-adrenergic agents
  4. Oral corticosteroids

21.  Which of the following treatment goals is best for the client with status asthmaticus?

  1. Avoiding intubation
  2. Determining the cause of the attack
  3. Improving exercise tolerance
  4. Reducing secretions

22.  Dani was given dilaudid for pain. She’s sleeping and her respiratory rate is 4 breaths/minute. If action isn’t taken quickly, she might have which of the following reactions?

  1. Asthma attack
  2. Respiratory arrest
  3. Be pissed about receiving Narcan
  4. Wake up on her own

23.  Which of the following additional assessment data should immediately be gathered to determine the status of a client with a respiratory rate of 4 breaths/minute?

  1. Arterial blood gas (ABG) and breath sounds
  2. Level of consciousness and a pulse oximetry value.
  3. Breath sounds and reflexes
  4. Pulse oximetry value and heart sounds

24.  A client is in danger of respiratory arrest following the administration of a narcotic analgesic. An arterial blood gas value is obtained. The nurse would expect to PaCO2 to be which of the following values?

  1. 15 mm Hg
  2. 30 mm Hg
  3. 40 mm Hg
  4. 80 mm Hg

25.  A client has started a new drug for hypertension. Thirty minutes after he takes the drug, he develops chest tightness and becomes short of breath and tachypneic. He has a decreased level of consciousness. These signs indicate which of the following conditions?

  1. Asthma attack
  2. Pulmonary embolism
  3. respiratory failure
  4. Rheumatoid arthritis

26.  Emergency treatment for a client with impending anaphylaxis secondary to hypersensitivity to a drug should include which of the following actions first?

  1. Administering oxygen
  2. Inserting an I.V. catheter
  3. Obtaining a complete blood count (CBC)
  4. Taking vital signs

27.  Following the initial care of a client with asthma and impending anaphylaxis from hypersensitivity to a drug, the nurse should take which of the following steps next?

  1. Administer beta-adrenergic blockers
  2. Administer bronchodilators
  3. Obtain serum electrolyte levels
  4. Have the client lie flat in the bed.

28.  A client’s ABG results are as follows: pH: 7.16; PaCO2 80 mm Hg; PaO2 46 mm Hg; HCO3– 24 mEq/L; SaO2 81%. This ABG result represents which of the following conditions?

  1. Metabolic acidosis
  2. Metabolic alkalosis
  3. Respiratory acidosis
  4. Respiratory alkalosis

29.  A nurse plans care for a client with chronic obstructive pulmonary disease, knowing that the client is most likely to experience what type of acid-base imbalance?

  1. Respiratory acidosis
  2. Respiratory alkalosis
  3. Metabolic acidosis
  4. Metabolic alkalosis

30.  A nurse is caring for a client who is on a mechanical ventilator. Blood gas results indicate a pH of 7.50 and a PCO2 of 30 mm Hg. The nurse has determined that the client is experiencing respiratory alkalosis. Which laboratory value would most likely be noted in this condition?

  1. Sodium level of 145 mEq/L
  2. Potassium level of 3.0 mEq/L
  3. Magnesium level of 2.0 mg/L
  4. Phosphorus level of 4.0 mg/dl

31.  A nurse reviews the arterial blood gas results of a patient and notes the following: pH 7.45; PCO2 30 mm Hg; and bicarbonate concentration of 22 mEq/L. The nurse analyzes these results as indicating:

  1. Metabolic acidosis, compensated.
  2. Metabolic alkalosis, uncompensated.
  3. Respiratory alkalosis, compensated.
  4. Respiratory acidosis, compensated.

32.  A client is scheduled for blood to be drawn from the radial artery for an ABG determination. Before the blood is drawn, an Allen’s test is performed to determine the adequacy of the:

  1. Popliteal circulation
  2. Ulnar circulation
  3. Femoral circulation
  4. Carotid circulation

33.  A nurse is caring for a client with a nasogastric tube that is attached to low suction. The nurse monitors the client, knowing that the client is at risk for which acid-base disorder?

  1. Respiratory acidosis
  2. Respiratory alkalosis
  3. Metabolic acidosis
  4. Metabolic alkalosis

34.  A nurse is caring for a client with an ileostomy understands that the client is most at risk for developing which acid-base disorder?

  1. Respiratory acidosis
  2. Respiratory alkalosis
  3. Metabolic acidosis
  4. Metabolic alkalosis

35.  A nurse is caring for a client with diabetic ketoacidosis and documents that the client is experiencing Kussmaul’s respirations. Based on this documentation, which of the following did the nurse observe?

  1. Respirations that are abnormally deep, regular, and increased in rate.
  2. Respirations that are regular but abnormally slow.
  3. Respirations that are labored and increased in depth and rate
  4. Respirations that cease for several seconds.

36.  A nurse understands that the excessive use of oral antacids containing bicarbonate can result in which acid-base disturbance?

  1. Respiratory alkalosis
  2. Respiratory acidosis
  3. Metabolic acidosis
  4. Metabolic alkalosis

37.  A nurse is caring for a client with renal failure. Blood gas results indicate a pH of 7.30; a PCO2 of 32 mm Hg, and a bicarbonate concentration of 20 mEq/L. The nurse has determined that the client is experiencing metabolic acidosis. Which of the following laboratory values would the nurse expect to note?

  1. Sodium level of 145 mEq/L
  2. Magnesium level of 2.0 mg/dL
  3. Potassium level of 5.2 mEq/L
  4. Phosphorus level of 4.0 mg/dL

38.  A nurse is preparing to obtain an arterial blood gas specimen from a client and plans to perform the Allen’s test on the client. Number in order of priority the steps for performing the Allen’s test (#1 is first step).

  1. Ask the client to open and close the hand repeatedly.
  2. Apply pressure over the ulnar and radial arteries.
  3. Assess the color of the extremity distal to the pressure point
  4. Release pressure from the ulnar artery
  5. Explain the procedure to the client.

39.  A nurse is preparing to obtain a sputum specimen from a client. Which of the following nursing actions will facilitate obtaining the specimen?

  1. Limiting fluids
  2. Having the client take 3 deep breaths.
  3. Asking the client to spit into the collection container.
  4. Asking the client to obtain the specimen after eating.

40.  A nurse is caring for a client after a bronchoscopy and biopsy. Which of the following signs if noted in the client should be reported immediately to the physician?

  1. Blood-streaked sputum
  2. Dry cough
  3. Hematuria
  4. Bronchospasm

41.  A nurse is suctioning fluids from a client via a tracheostomy tube. When suctioning, the nurse must limit the suctioning to a maximum of:

  1. 5 seconds
  2. 10 seconds
  3. 30 seconds
  4. 1 minute

42.  A nurse is suctioning fluids from a client through an endotracheal tube. During the suctioning procedure, the nurse notes on the monitor that the heart rate decreases. Which of the following is the most appropriate nursing intervention?

  1. Continue to suction
  2. Ensure that the suction is limited to 15 seconds
  3. Stop the procedure and reoxyenate the client
  4. Notify the physician immediately.

43.  An unconscious client is admitted to an emergency room. Arterial blood gas measurements reveal a pH of 7.30, a low bicarbonate level, a normal carbon dioxide level, and a normal oxygen level. An elevated potassium level is also present. These results indicate the presence of:

  1. Metabolic acidosis
  2. Respiratory acidosis
  3. Combined respiratory and metabolic acidosis
  4. overcompensated respiratory acidosis

44.  A nurse is caring for a client hospitalized with acute exacerbation of COPD. Which of the following would the nurse expect to note on assessment of this client?

  1. Increased oxygen saturation with exercise
  2. Hypocapnia
  3. A hyperinflated chest on x-ray film
  4. A widened diaphragm noted on chest x-ray film

45.  An oxygenated delivery system is prescribed for a client with COPD to deliver a precise oxygen concentration. Which of the following types of oxygen delivery systems would the nurse anticipate to be prescribed?

  1. Venturi mask
  2. Aerosol mask
  3. Face tent
  4. Tracheostomy collar

46.  Theophylline (Theo-Dur) tablets are prescribed for a client with chronic airflow limitation, and the nurse instructs the client about the medication. Which statement by the client indicates a need for further teaching?

  1. “I will take the medication on an empty stomach.”
  2. “I will take the medication with food.”
  3. “I will continue to take the medication even if I am feeling better.”
  4. “Periodic blood levels will need to be obtained.”

47.  A nurse is caring for a client with emphysema. The client is receiving oxygen. The nurse assesses the oxygen flow rate to ensure that it does not exceed

  1. 1 L/min
  2. 2 L/min
  3. 6 L/min
  4. 10 L/min

48.  The nurse reviews the ABG values of a client. The results indicate respiratory acidosis. Which of the following values would indicate that this acid-base imbalance exists?

  1. pH of 7.48
  2. PCO2 of 32 mm Hg
  3. pH of 7.30
  4. HCO3– of 20 mEq/L

49.  A nurse instructs a client to use the pursed lip method of breathing. The client asks the nurse about the purpose of this type of breathing. The nurse responds, knowing that the primary purpose of pursed lip breathing is:

  1. Promote oxygen intake
  2. Strengthen the diaphragm
  3. Strengthen the intercostal muscles
  4. Promote carbon dioxide elimination

50.  A nurse reviews the ABG values and notes a pH of 7.50, a PCO2 of 30 mm Hg, and an HCO3 of 25 mEq/L. The nurse interprets these values as indicating:

  1. Respiratory acidosis uncompensated
  2. Respiratory alkalosis uncompensated
  3. Metabolic acidosis uncompensated
  4. Metabolic acidosis partially compensated.

51.  Aminophylline (theophylline) is prescribed for a client with acute bronchitis. A nurse administers the medication, knowing that the primary action of this medication is to:

  1. Promote expectoration
  2. Suppress the cough
  3. Relax smooth muscles of the bronchial airway
  4. Prevent infection

52.  A client is receiving isoetharine hydrochloride (Bronkosol) via a nebulizer. The nurse monitors the client for which side effect of this medication?

  1. Constipation
  2. Diarrhea
  3. Bradycardia
  4. Tachycardia

53.  A nurse teaches a client about the use of a respiratory inhaler. Which action by the client indicated a need for further teaching?

  1. Removes the cap and shakes the inhaler well before use.
  2. Presses the canister down with finger as he breathes in.
  3. Inhales the mist and quickly exhales.
  4. Waits 1 to 2 minutes between puffs if more than one puff has been prescribed.

54.  A female client is scheduled to have a chest radiograph. Which of the following questions is of most importance to the nurse assessing this client?

  1. “Is there any possibility that you could be pregnant?”
  2. “Are you wearing any metal chains or jewelry?”
  3. “Can you hold your breath easily?”
  4. “Are you able to hold your arms above your head?”

55.  A client has just returned to a nursing unit following bronchoscopy. A nurse would implement which of the following nursing interventions for this client?

  1. Encouraging additional fluids for the next 24 hours
  2. Ensuring the return of the gag reflex before offering foods or fluids
  3. Administering atropine intravenously
  4. Administering small doses of midazolam (Versed).

56.  A client has an order to have radial ABG drawn. Before drawing the sample, a nurse occludes the:

  1. Brachial and radial arteries, and then releases them and observes the circulation of the hand.
  2. Radial and ulnar arteries, releases one, evaluates the color of the hand, and repeats the process with the other artery.
  3. Radial artery and observes for color changes in the affected hand.
  4. Ulnar artery and observes for color changes in the affected hand.

57.  A nurse is assessing a client with chronic airflow limitation and notes that the client has a “barrel chest.” The nurse interprets that this client has which of the following forms of chronic airflow limitation?

  1. Chronic obstructive bronchitis
  2. Emphysema
  3. Bronchial asthma
  4. Bronchial asthma and bronchitis

58.  A client has been taking benzonatate (Tessalin Perles) as prescribed. A nurse concludes that the medication is having the intended effect if the client experiences:

  1. Decreased anxiety level
  2. Increased comfort level
  3. Reduction of N/V
  4. Decreased frequency and intensity of cough

59.  Which of the following would be an expected outcome for a client recovering from an upper respiratory tract infection? The client will:

  1. Maintain a fluid intake of 800ml every 24 hours.
  2. Experience chills only once a day
  3. Cough productively without chest discomfort.
  4. Experience less nasal obstruction and discharge.

60.  Which of the following individuals would the nurse consider to have the highest priority for receiving an influenza vaccination?

  1. A 60-year-old man with a hiatal hernia
  2. A 36-year-old woman with 3 children
  3. A 50-year-old woman caring for a spouse with cancer
  4. a 60-year-old woman with osteoarthritis

61.  A client with allergic rhinitis asks the nurse what he should do to decrease his symptoms. Which of the following instructions would be appropriate for the nurse to give the client?

  1. “Use your nasal decongestant spray regularly to help clear your nasal passages.”
  2. “Ask the doctor for antibiotics. Antibiotics will help decrease the secretion.”
  3. “It is important to increase your activity. A daily brisk walk will help promote drainage.”
  4. “Keep a diary if when your symptoms occur. This can help you identify what precipitates your attacks.”

62.  An elderly client has been ill with the flu, experiencing headache, fever, and chills. After 3 days, she develops a cough productive of yellow sputum. The nurse auscultates her lungs and hears diffuse crackles. How would the nurse best interpret these assessment findings?

  1. It is likely that the client is developing a secondary bacterial pneumonia.
  2. The assessment findings are consistent with influenza and are to be expected.
  3. The client is getting dehydrated and needs to increase her fluid intake to decrease secretions.
  4. The client has not been taking her decongestants and bronchodilators as prescribed.

63.  Guaifenesin 300 mg four times daily has been ordered as an expectorant. The dosage strength of the liquid is 200mg/5ml. How many mL should the nurse administer each dose?

  1. 5.0 ml
  2. 7.5 ml
  3. 9.5 ml
  4. 10 ml

64.  Pseudoephedrine (Sudafed) has been ordered as a nasal decongestant. Which of the following is a possible side effect of this drug?

  1. Constipation
  2. Bradycardia
  3. Diplopia
  4. Restlessness

65.  A client with COPD reports steady weight loss and being “too tired from just breathing to eat.” Which of the following nursing diagnoses would be most appropriate when planning nutritional interventions for this client?

  1. Altered nutrition: Less than body requirements related to fatigue.
  2. Activity intolerance related to dyspnea.
  3. Weight loss related to COPD.
  4. Ineffective breathing pattern related to alveolar hypoventilation.

66.  When developing a discharge plan to manage the care of a client with COPD, the nurse should anticipate that the client will do which of the following?

  1. Develop infections easily
  2. Maintain current status
  3. Require less supplemental oxygen
  4. Show permanent improvement.

67.  Which of the following outcomes would be appropriate for a client with COPD who has been discharged to home? The client:

  1. Promises to do pursed lip breathing at home.
  2. States actions to reduce pain.
  3. States that he will use oxygen via a nasal cannula at 5 L/minute.
  4. Agrees to call the physician if dyspnea on exertion increases.

68.  Which of the following physical assessment findings would the nurse expect to find in a client with advanced COPD?

  1. Increased anteroposterior chest diameter
  2. Underdeveloped neck muscles
  3. Collapsed neck veins
  4. Increased chest excursions with respiration

69.  Which of the following is the primary reason to teach pursed-lip breathing to clients with emphysema?

  1. To promote oxygen intake
  2. To strengthen the diaphragm
  3. To strengthen the intercostal muscles
  4. To promote carbon dioxide elimination

70.  Which of the following is a priority goal for the client with COPD?

  1. Maintaining functional ability
  2. Minimizing chest pain
  3. Increasing carbon dioxide levels in the blood
  4. Treating infectious agents

71.  A client’s arterial blood gas levels are as follows: pH 7.31; PaO2 80 mm Hg, PaCO2 65 mm Hg; HCO3– 36 mEq/L. Which of the following signs or symptoms would the nurse expect?

  1. Cyanosis
  2. Flushed skin
  3. Irritability
  4. Anxiety

72.  When teaching a client with COPD to conserve energy, the nurse should teach the client to lift objects:

  1. While inhaling through an open mouth.
  2. While exhaling through pursed lips
  3. After exhaling but before inhaling.
  4. While taking a deep breath and holding it.

73.  The nurse teaches a client with COPD to assess for s/s of right-sided heart failure. Which of the following s/s would be included in the teaching plan?

  1. Clubbing of nail beds
  2. Hypertension
  3. Peripheral edema
  4. Increased appetite

74.  The nurse assesses the respiratory status of a client who is experiencing an exacerbation of COPD secondary to an upper respiratory tract infection. Which of the following findings would be expected?

  1. Normal breath sounds
  2. Prolonged inspiration
  3. Normal chest movement
  4. Coarse crackles and rhonchi

75.  Which of the following ABG abnormalities should the nurse anticipate in a client with advanced COPD?

  1. Increased PaCO2
  2. Increased PaO2
  3. Increased pH.
  4. Increased oxygen saturation

76.  Which of the following diets would be most appropriate for a client with COPD?

  1. Low fat, low cholesterol
  2. Bland, soft diet
  3. Low-Sodium diet
  4. High calorie, high-protein diet

77.  The nurse is planning to teach a client with COPD how to cough effectively. Which of the following instructions should be included?

  1. Take a deep abdominal breath, bend forward, and cough 3 to 4 times on exhalation.
  2. Lie flat on back, splint the thorax, take two deep breaths and cough.
  3. Take several rapid, shallow breaths and then cough forcefully.
  4. Assume a side-lying position, extend the arm over the head, and alternate deep breathing with coughing.

78.  A 34-year-old woman with a history of asthma is admitted to the emergency department. The nurse notes that the client is dyspneic, with a respiratory rate of 35 breaths/minute, nasal flaring, and use of accessory muscles. Auscultation of the lung fields reveals greatly diminished breath sounds. Based on these findings, what action should the nurse take to initiate care of the client?

  1. Initiate oxygen therapy and reassess the client in 10 minutes.
  2. Draw blood for an ABG analysis and send the client for a chest x-ray.
  3. Encourage the client to relax and breathe slowly through the mouth
  4. Administer bronchodilators

79.  The nurse would anticipate which of the following ABG results in a client experiencing a prolonged, severe asthma attack?

  1. Decreased PaCO2, increased PaO2, and decreased pH.
  2. Increased PaCO2, decreased PaO2, and decreased pH.
  3. Increased PaCO2, increased PaO2, and increased pH.
  4. Decreased PaCO2, decreased PaO2, and increased pH.

80.  A client with acute asthma is prescribed short-term corticosteroid therapy. What is the rationale for the use of steroids in clients with asthma?

  1. Corticosteroids promote bronchodilation
  2. Corticosteroids act as an expectorant
  3. Corticosteroids have an anti-inflammatory effect
  4. Corticosteroids prevent development of respiratory infections.

81.  The nurse is teaching the client how to use a metered dose inhaler (MDI) to administer a Corticosteroid drug. Which of the following client actions indicates that he us using the MDI correctly? Select all that apply.

  1. The inhaler is held upright.
  2. Head is tilted down while inhaling the medication
  3. Client waits 5 minutes between puffs.
  4. Mouth is rinsed with water following administration
  5. Client lies supine for 15 minutes following administration.

82.  A client is prescribed metaproterenol (Alupent) via a metered dose inhaler (MDI), two puffs every 4 hours. The nurse instructs the client to report side effects. Which of the following are potential side effects of metaproterenol?

  1. Irregular heartbeat
  2. Constipation
  3. Petal edema
  4. Decreased heart rate.

83.  A client has been taking flunisolide (Aerobid), two inhalations a day, for treatment of asthma. He tells the nurse that he has painful, white patches in his mouth. Which response by the nurse would be the most appropriate?

  1. “This is an anticipated side-effect of your medication. It should go away in a couple of weeks.”
  2. “You are using your inhaler too much and it has irritated your mouth.”
  3. “You have developed a fungal infection from your medication. It will need to be treated with an antibiotic.”
  4. “Be sure to brush your teeth and floss daily. Good oral hygiene will treat this problem.”

84.  Which of the following health promotion activities should the nurse include in the discharge teaching plan for a client with asthma?

  1. Incorporate physical exercise as tolerated into the treatment plan.
  2. Monitor peak flow numbers after meals and at bedtime.
  3. Eliminate stressors in the work and home environment
  4. Use sedatives to ensure uninterrupted sleep at night.

85.  The client with asthma should be taught that which of the following is one of the most common precipitating factors of an acute asthma attack?

  1. Occupational exposure to toxins
  2. Viral respiratory infections
  3. Exposure to cigarette smoke
  4. Exercising in cold temperatures

86.  A female client comes into the emergency room complaining of SOB and pain in the lung area. She states that she started taking birth control pills 3 weeks ago and that she smokes. Her VS are: 140/80, P 110, R 40. The physician orders ABG’s, results are as follows: pH: 7.50; PaCO2 29 mm Hg; PaO2 60 mm Hg; HCO3– 24 mEq/L; SaO2 86%. Considering these results, the first intervention is to:

  1. Begin mechanical ventilation
  2. Place the client on oxygen
  3. Give the client sodium bicarbonate
  4. Monitor for pulmonary embolism.

87.  Basilar crackles are present in a client’s lungs on auscultation. The nurse knows that these are discrete, noncontinuous sounds that are:

  1. Caused by the sudden opening of alveoli
  2. Usually more prominent during expiration
  3. Produced by airflow across passages narrowed by secretions
  4. Found primarily in the pleura.

88.  A cyanotic client with an unknown diagnosis is admitted to the E.R. In relation to oxygen, the first nursing action would be to:

  1. Wait until the client’s lab work is done.
  2. Not administer oxygen unless ordered by the physician.
  3. Administer oxygen at 2 L flow per minute.
  4. Administer oxygen at 10 L flow per minute and check the client’s nailbeds.

89.  Immediately following a thoeacentesis, which clinical manifestations indicate that a complication has occurred and the physician should be notified?

  1. Serosanguineous drainage from the puncture site
  2. Increased temperature and blood pressure
  3. Increased pulse and pallor
  4. Hypotension and hypothermia

90.  If a client continues to hypoventilate, the nurse will continually assess for a complication of:

  1. Respiratory acidosis
  2. Respiratory alkalosis
  3. Metabolic acidosis
  4. Metabolic alkalosis

91.  A client is admitted to the hospital with acute bronchitis. While taking the client’s VS, the nurse notices he has an irregular pulse. The nurse understands that cardiac arrhythmias in chronic respiratory distress are usually the result of:

  1. Respiratory acidosis
  2. A build-up of carbon dioxide
  3. A build-up of oxygen without adequate expelling of carbon dioxide.
  4. An acute respiratory infection.

92.  Auscultation of a client’s lungs reveals crackles in the left posterior base. The nursing intervention is to:

  1. Repeat auscultation after asking the client to deep breathe and cough.
  2. Instruct the client to limit fluid intake to less than 2000 ml/day.
  3. Inspect the client’s ankles and sacrum for the presence of edema
  4. Place the client on bedrest in a semi-Fowlers position.

93.  The most reliable index to determine the respiratory status of a client is to:

  1. Observe the chest rising and falling
  2. Observe the skin and mucous membrane color.
  3. Listen and feel the air movement.
  4. Determine the presence of a femoral pulse.

94.  A client with COPD has developed secondary polycythemia. Which nursing diagnosis would be included in the plan of care because of the polycythemia?

  1. Fluid volume deficit related to blood loss.
  2. Impaired tissue perfusion related to thrombosis
  3. Activity intolerance related to dyspnea
  4. Risk for infection related to suppressed immune response.

95.  The physician has scheduled a client for a left pneumonectomy. The position that will most likely be ordered postoperatively for his is the:

  1. Unoperative side or back
  2. Operative side or back
  3. Back only
  4. Back or either side.

96.  Assessing a client who has developed atelectasis postoperatively, the nurse will most likely find:

  1. A flushed face
  2. Dyspnea and pain
  3. Decreased temperature
  4. Severe cough and no pain.

97.  A fifty-year-old client has a tracheostomy and requires tracheal suctioning. The first intervention in completing this procedure would be to:

  1. Change the tracheostomy dressing
  2. Provide humidity with a trach mask
  3. Apply oral or nasal suction
  4. Deflate the tracheal cuff

98.  A client states that the physician said the tidal volume is slightly diminished and asks the nurse what this means. The nurse explains that the tidal volume is the amount of air:

  1. Exhaled forcibly after a normal expiration
  2. Exhaled after there is a normal inspiration
  3. Trapped in the alveoli that cannot be exhaled
  4. Forcibly inspired over and above a normal respiration.

99.  An acceleration in oxygen dissociation from hemoglobin, and thus oxygen delivery to the tissues, is caused by:

  1. A decreasing oxygen pressure in the blood
  2. An increasing carbon dioxide pressure in the blood
  3. A decreasing oxygen pressure and/or an increasing carbon dioxide pressure in the blood.
  4. An increasing oxygen pressure and/or a decreasing carbon dioxide pressure in the blood.

100. Lisa is newly diagnosed with asthma and is being discharged from the hospital after an episode of status asthmaticus. Discharge teaching should include which of the following:

What is the most reliable index to determine the respiratory status of a client is to?

Answer: 3. To check for breathing, the nurse places her ear and cheek next to the client's mouth and nose to listen and feel for air movement. The chest rising and falling (1) is not conclusive of a patent airway.

What is the best way to assess a client's respiratory rate and pattern?

Thus, the nurse should not tell the client to breathe normally or deeply. Using a watch with a second hand, count the number of respirations for 30 seconds. Multiply this number by 2 to calculate the respiratory rate per minute.

What parameters should be checked when assessing respiration?

In particular, three parameters of respiration patterns are evaluated: the respiration rate for normal and abnormal breathing patterns, the presence of breathing pauses or apnea events (cessation of respiration) and the relationship between respiration effort and respiration activity (respiration depth).

What is a focused respiratory assessment?

A focused respiratory system assessment includes collecting subjective data about the patient's history of smoking, collecting the patient's and patient's family's history of pulmonary disease, and asking the patient about any signs and symptoms of pulmonary disease, such as cough and shortness of breath.